Download as docx, pdf, or txt
Download as docx, pdf, or txt
You are on page 1of 136

1

LAWS3306 Study Guide:

Models of the Criminal Process


1. The Criminal Process
a. Definition
 It describes the activities and actors which respond to crime
b. Inverted Triangle: the criminal justice process acts as a funnel, the funnel gets
smaller and smaller which results in cases that end up going to court, resulting in
acquittals
 Top unreported crime
 Middle crime that doesn’t make it to court
 Bottom (the tip) not many cases go through to this stage (trial)
c. Expenditures in the system: the majority of money is spent reactively, and less-so
on proactive measures
 Division
- Policing  the largest share is spent on policing
- Corrections  the second most is spent on corrections (jails,
penitentiaries, parole, and probation)
- Billings  this includes the court staff, legal clerks, judges etc.
- Legal Aid  the smallest share is given to people who don’t have
money to hire a lawyer
 Funding and crime stats
- the current spending = 1.1% of nominal domestic product which
means that 20B of the overall GDP is spent on the criminal process
in Canada
a. about the same amount is allocated and spent on national
defense, and the amount is about double of that of
Northern and Indigenous Affairs
- Spending increase
a. The expenditures have increased despite overall crime rates
and violent crime being on the decline since the 60s’ and
70s’
i. In 2002, our spending on the criminal justice
system increased by 23% until 2011, within this
span, our crime rate has declined by 23% during
that same period
- Crime rates, spending and the media
a. 1.1% of spending attracts a lot of attention from the media
which has resulted in media outlets reporting criminal
related matters and talking about crime often in national
discourse
2

i. During the previous federal election, the


conservative government campaigned and ran on
the premise of being “tough on crime” despite
overall crime and violent crime being on the
decrease
b. Media and discourse on crime = “the criminalization of
politics”
i. Our policies on social justice issues are often
looked at through the lens of criminal law. Society
and communities may define social justice issues
and thus the CJS narrows this into criminal law or
amendments to this law as a means to fix the issue
ii. Importance  the criminalization of politics and
where we can see this in action (mandatory
minimums in the wake of the conservative’s “tough
on crime” while Harper was in term)
2. Models – Issues
a. Restrictions
 Its not possible to reduce our criminal justice system and actors to a
single truth and theory/ model; not only theory is the be-all-end-all and
must be taken into context and perspective. It is, however, valuable to
identify the areas where each model is dominant to have a sense for
overall trends and tendencies which are prevalent within the criminal
justice system, policing, courts, and laws

3. Packer’s Two Models:


a. Crime Control
 Metaphor assembly line/ conveyor belt
- this represents the goal of this model of efficiency, the model aims
to move an individual as quickly as possible from factual to legal
guilt; this is accomplished with minimal interference/ checks-and -
balances which would slow the system.
 Concern  System efficiency and victims rights
- Goal from factual to legal guilt as quickly as possible is done
through a) method of how they act as gate keepers= through fact
finding process, b) simply concerned with reasonable probable
grounds standard, not beyond a reasonable doubt.
 Validating authority  the legislature (Parliament), not the Courts
 The criminal sanction  positive
- There is a heavy reliance on the criminal sanction, punishment is
viewed as being a positive factor, and seen as necessary to
maintain public order in society
3

- The sanction is important because its employed to protect people,


property and promote order in society which can lead to social
stability This is the premise of deterrence practices and the belief
that people will not commit criminal offences if the threat of
punishment is greater.
- Pre-sentenced custody is used as a coercive method to get
someone form factual to legal guilt as fast as possible
 Fact-finding  front line actors
- The front-line actors which this model utilizes are the police (they
make judgement of who is involved in the first place), the Crown
Attorney is seen as the acting gatekeeper.
 Investigative powers  given to the police to arrest people for
questioning, this is the quickest method to establish if a suspect is innocent
or not
- They are given broad discretionary powers which are used to find
the quickest means by which to establish the factual guilt of the
accused
a. Example  illegally seized evidence ought to be used in
trial
 The trial not important
- The trial is not important in the CC model because the centre of
gravity lies in the early administrative fact-finding stages
a. Pre-trial detention is important to ensure future crime by
the accused will not continue to occur and to persuade the
accused to plead.
b. Guilty pleas are important to ensure system efficiency and
are encouraged via sentencing discounts for pleading early
for saving resources
c. Decided on a balance of probabilities, not beyond a
reasonable doubt
 Appeals not to be encouraged
- This would result in system in-efficiency and should not be
encouraged
a. They should only be allowed if the accused establishes that
o reasonable tier of fact could have convicted on the
evidence presented
 Limitation  police powers and minimal use of checks and balances
- The limitation lies in the overly broad discretion given to police in
investigation in which evidence, even if improperly collected can
be admitted and allows them to ensure evidence does not get
trained, methods such as torture may be used.
- Minimal use of checks and balances which allow police to use
investigative methodologies as they wish, meaning that because of
4

the overly expedited process can result in innocent people behind


bars
a. However, CC asserts that the acquittal of 1 guilty
individual is worse than the conviction of 10 innocent
people
- Largest enemy = anything that obstructs, hinders, etc. the
prosecution.
 Misconduct  handled outside of the formal system
- Taken very seriously, no one is above the law
b. Due Process
 Metaphor  obstacle course
- The obstacle course represents the non-linear process that is the
criminal justice system and how many checks and balances are in
place to ensure the rights of the victim, accused are weighed to
ensure the system of justice is not thrown into disrepute
 Concern  fairness and rights of the accused
- Concerned with the fairness of the process and the intrusiveness of
the state in our everyday lives and how the state is bound to
respect our civil liberties
 Validating authority  The courts
- Believes that the Courts are the validating authority. It believes
that the focus at the top of efficiency is equal treatment and that
through equal treatment can be used to ensure representation for
marginalized individuals via legal aid plan and public defenders
through actual procedures. The belief here is that the when the
basic due process rights are protected within the system, you are
also protecting those who are the most marginalized.
 The criminal sanction  negative and skeptical
- The criminal sanction here is seen as negative, and the model
posits that instead, there should be a reliance on de-
criminalization to reduce the workload of the criminal justice
system and as a way to aim resources to more serious criminal
activity
- Pre-sentenced custody as a coercive method
 Fact-finding  minimization of informal fact finding
- The fact finding should be conducted legally and ought to protect
the rights of the suspects and the minimization of informal fact-
finding in streets and stationhouses
a. Police shouldn’t detain an individual just to develop their
case
b. Accused must be informed of the right to remain silent
 Investigative powers  broad discretionary powers are limited
5

- These are limited to ensure the checks and balances are followed
and not overridden for the sake of developing a case
 The trial  important
- The DP model asserts that in the trial, the concern is based in
whether the prosecution can establish beyond a reasonable doubt
on the basis of legally obtained evidence
- Only defense lawyers and appointed judges can be relied upon to
appreciate the importance of legal guilt, not the police
- Not seen as a burden and is a logical and proper cumulation of the
process
 Appeals  important
- Due to concerns about even the minor risks of convicting an
innocent individual, the accused should have wide rights to appeal
and the appellant courts should reverse convictions whenever trial
judges fail to protect the accused’s rights
a. The reversal of convictions = small price to pay for an
affirmation of proper values are sighted
b. SCC = imp because it defines the legal rights and remedies
of the accused
 Limitation  slow
- The main limitation of this model is that because of delays created
in the obstacle course, however, it is balanced by the fact that they
ensure to their best capability that innocent people are not placed
behind bars
a. This is held as very important in this model
 Misconduct handled within the system
- Evidence collected inappropriately cannot be used against you
and if a state actor is engaged in investigatory techniques which
can violate civil liberties, they can’t use it later on you and have to
prosecute and investigate via existing guidelines and frameworks
- Heavy reliance on guilty beyond a reasonable don’t and innocent
until proven guilty
c. Context  the models were developed y Packer to describe the criminal process
during the 60s’
 The US criminal justice system
- The Warren Era
a. relevant time in the US criminal justice and process
because of the increase in judicial act and increase in
desire to protect persons. In the 60s’ we had what was the
Warren Court enacted many due-process procedures which
aimed to protect the rights of the accused and victims
through procedural fairness
6

i. the creation of exclusionary rules (Mapp 1961) 


in terms of evidence and procedure to exclude
evidence if found in violation of a person’s civil
liberties and greater regulation of police powers
ii. right to defense council (Gideon v Wainwright) 
requires the states to provide defense counsel to
those charged with felonies who could not afford to
hire their own lawyer *most noteworthy
accomplishment*
iii. development of Miranda rights (Miranda 1966) 
ruling that police must inform their suspects of their
right to counsel, including publicly funded counsel.
Failure to provide these warnings or allow an
accused to obtain a lawyer would result in the
exclusion of even trustworthy confessions
iv. disclosure of exculpatory and inculpatory evidence
to the accused (Brady 1963)
v. accused’s silence could not be used against them
(Griffin 1965)
vi. belief that young persons had to be treated
differently than adults (Gault 1967)  that more
robust measures must be in place for protections
from the state
1. state and powers of that vs a youth
vii. right to have guilt proven beyond a reasonable
doubt (Klapfer 1967)
b. Fragility of the DP model
i. The model and system are fragile and require
constant attention to ensure that the actors must
conduct themselves and how even minor changes
can cause major differences in the justice system
ii. One thing that occurred before Packer died was the
belief of DP as a concept and rights of an accused
person and rights of them in re: to state
interference
1. Failure
2. Saying that the Warren court really did
nothing in regard to how people were dealt
with and were processed through the
criminal justice system; most symbolic than
anything
3. Heavier de-criminalization ought to have
been implemented
7

4. These models were developed prior to the


rise of feminism in the US through the 70s’
 The Canadian criminal justice system
- Development of the Bill of Rights by Diefenbaker (pre- Charter)
a. Enshrined many rights and procedural aspects outlined in
many decisions made by the Warren government
4. Critics (4 critiques)
a. Empirical irrelevance
 Defense lawyer’s vs everyone else
- There is a belief that the interest of the defense or defense counsel
is always contrary to that of the state or the process/ other actors of
the process when in reality, empirical studies done on the criminal
justice system reveal that all actors, no matter if the Crown, Judge
etc. all share common goals.
a. the view that it’s the defense vs everyone else when in
reality all actors cooperate with each other to ensure that
the system runs smoothly; this in contrary to the notion that
the defense counsel is bad
 the cost
- to an accused person, evoking their right to due process right is
often times greater than the value of the right itself.
a. “do I plead guilty or do I want to wait months to go to
trial”
i. To even prove that their civil rights have been
violated, they are going to have sit in custody for a
long time. It connects the cost of evoking this right
is worth more than the right itself is worth to plead
guilty than to fight
ii. These principles are only significant when you deal
with big cases where these due process rights are
actually applied
iii. Belief that defense lawyers are agents of crime
control and that they find guilty pleas to be in their
best interest
b. Due process = crime control?
 Based on the fact that formal laws are developed and formed via
Parliament and court decisions and in some of these decisions and laws,
they enable police and prosecutors to exercise broad discretionary powers
- Prior to the enactment of the Charter, the limits on police were
silent and no set defined boundaries of police limits, therefore, they
went beyond boundaries because the courts lacks the tools to hold
them accountable in this fashion
8

- That due process or having a system in due process in nature helps


with the imposition of sanctions
a. The CC model relies heavily on the sanction, whereas, the
DP model maintains that the sanction doesn’t do much for
society
b. Critics say that when you have a DP model system and get
someone from factual to legal guilt, that society doesn’t
become concerned anymore for the criminal sanction as it
is seen as being a fair system in place
i. This it legitimizes punishment as so long as they are
found guilty beyond a reasonable doubt
1. Illusion of DP, when really its still a
conveyor belt and only legitimizes the
sentence
2. After the Charter was enacted, resulted in
the large recognition of civil liberties and
the recognition to protect these liberties and
that if these rights are breached, there must
be punishment (exclusion of evidence if
collected improperly)
c. In theory, through the recognition of these protections and
enactment of these methods in violations of rights, that
your prison populations would decrease as you’d enact
exclusionary tools and rights. In Canada, we see the
opposite
i. Post- Charter, prison population hasn’t gone down,
but rather has increased
ii. Critics say that DP acts as a veneer of fairness in a
system that has an increasing punishments of
individuals that is legitimized because only the
guilty are placed behind bars
d. The role of the media and public consumption of the
criminal justice system
i. Important SC decisions not on front pages of news
1. Problem = offensive DP rulings = person
accused of heinous offences and improperly
collected evidence. How the media doesn’t
acknowledge the DP process and describe a
narrative that a person has gotten off on a
technicality and civil liberties and rights are
regarded as such
c. Limited liberal and adversarial vision
9

 These models were developed in the US in the 60s and the necessary
acknowledgement that these models are limited in applicability to non-
adversarial justice systems
- Canada = adversarial (Crown, Judge, Neutral/ Independent judge
and or Arbitrator) vs Inquisitorial system where the court is
actively involved in the investigation of facts
a. The models don’t have much application of weight in the
adversarial system
b. Concept of a fight is typically seen as a negative check on
government, and Packer didn’t envision that is was a
positive guarantor of state action
c. The assumption that individuals have interests that were
always opposed to the interests of the state
i. Arms length rules = rooted in a system where we
are distrustful of state activity and involvement and
individuals not sharing the same interests and
common goals
d. Victims? (most importantly and exclusion of victims) *both CC and DP talk
about the state’s interest in the accused’s interest, nothing about the interest of
the victim*
 Lack of consideration of the victim and their role in the development of
these two models
- The crime funnel and importance of recognizing reported crime
a. Significant amount of unreported crime in Canadian
society. One of the critiques of Packer’s theory is why
wouldn’t he consider the lack of reported crime by victims
and need in subsequent models and the role that victims
play in the criminal system but in the way that laws are
created or changed and the way the process is changed
and or amended
i. Recent surveys in Canada illustrating that there’s a
large segment of the population who don’t engage
in the criminal justice process
1. Some people believe that people must
always report crime and no reason why
crime ought to go unreported in society, and
if there’s a high level of unreported crime, it
can be indicative of inadequacy within our
system to deal with this and some kind of
dissatisfaction with the process because it
assumes crime is a zero-sung game and that
once crime, occurs, the state should be
involved if there’s any criminal activity
10

ii. Recognition as it pertains to unreported crime that


legal pluralism has been created because of
unreported crime
1. Individuals have made their decision to take
matters into their own hands and not
necessarily a bad thing and people believing
this is a good thing and giving people the
autonomy to choice and partake in the
system and how this choice ought to be
respected
iii. Creation of a risk-based society we see develop
because of the recognition of victims and
victimization
1. This means that we have attempted to
minimize the risk of becoming a victim by
calculating these risks via developing our
criminal process (via laws and evidence)
with an eye towards victimization
minimization
2. We’ve become so focused with the concept
of risk and victimization that we’ve
developed mandatory minimums and
changed process in a way all to minimize
the risk of becoming a victim
3. Various groups using disproportionate
measures to document risks and make
demands for more changes
- Critique  that Packer missed the mark and the impact of the role
of victims and victimization held in the process

5. New Models:
a. Punitive Model
 This model can be represented via roller coaster
- With a linear line with ups and downs; linear representing crime
control and non-linear lines representing due process, there is a
focus here to a criminal sanction, similar to the CC model
a. what this model aims to explain is that the criminal justice
system is a battle of rights between that of the accused
against the rights of victims and potential victims.
i. Not victims, but those who are at an increased risk
of becoming victims
11

ii. They see the criminal justice system stuck between


competing interests and borrows a lot from the CC
model
b. Focus  equal protection
i. High level of unreported crime under this model is
a result of the failure of the justice system and there
should not be any unreported crime present in
society
c. How it differs from the CC model  deference to police
and the Crown
i. CC deference given to police and Crown to ensure
rights of people are involved in the process and to
weed out innocent people being found guilty
ii. These actors are subject to greater scrutiny
because they need to abide by victims’ rights and
how charges get dropped = no justice for victims
iii. If police do things improperly where evidence is
excluded, the victim does not receive justice
iv. Police activity and the Crown activities are treated
with great scrutiny to ensure they don’t go beyond
the powers that they’ve been given
d. The model takes issue with the CC model of encouraging
guilty pleas
i. The model takes issue with this because plea
bargaining does not involve the victim and is only
in the Crown’s responsibility to engage in plea
bargaining with the person involved
ii. Those interests sometimes collide with the Crown’s
interest of efficiency via giving favorable outcomes
iii. Early parts of this model also concerned with the
fact that there was no input with victims with plea
bargaining
1. Conservative gov and their Victim’s Bill of
Rights
2. Overtime victims playing a more active role
in the process in plea bargaining
3. The Crown is now bound to get input from
victims and the risk of victimization
e. This model focuses on factual guilt once found, just a
matter of time before legal guilt is determined beyond a
reasonable doubt
f. Concept of restorative justice which almost is extra-
judicial in nature where there is a reliance on activities
12

outside of formal activities dealing with punishment of


acts, this notion is rejected
i. Victim punitive models believe that once taking
concern for the accused, you may end up re-
victimizing the victim
ii. Increased victim presence in police and how they
operate
iii. Fierce opposition from DP because its seen as
diverting the factual guilt away from the individual
and that victims’ rights must be respected as much
as that of the accused
- This model challenges the concept of victimless crimes
a. Packer: if you have a consensual transaction between 2
people, there is an absence of a victim
i. The question is if this is right to see if there is a
victim or not
1. Victim model takes greater trouble with this
and the model calls for a greater expansion
of the meaning of harm
a. Classical definition of harm 
physical harm that is being caused
to an individual and what this model
aims to do is expand on what harm
is and not just limited to physical
realm of harm, but also tired to
notions of psychological harm
(abortion and assisted dying and
how psychological harm was used to
decriminalize these activities)
- Reducing risks of victimization
a. Increase in demand for state activism and calls for greater
protections
b. Asking state to become more active in investigating laws
etc.
c. Demands for increased state activism = further
criminalization of politics and focus on dealing with those
social issues through the lens of critical as opposed to the
issue and the cause of issue
d. How this is being used to justify these approaches and the
criminal sanctions
b. Non- Punitive Model
 This model can be characterized as a continually rotating circle
13

- One side is representative of the commission of the offence and


trying to get all parties back to where they were before the act was
committed
a. Non-punitive victims’ rights model is focused on being
proactive and not reactive, using as many restorative
justice steps as possible and restoring everyone to where
they were before
i. Victim punitive = reactive (sanction and
punishment vs Victim non-punitive = crime
prevention from the first place
1. in the circle, you recognize and depend
upon concepts such as health, compassion,
and restorative techniques to heal both the
accused and the victim
2. the punitive model and issue to unreported
crime a dhow the punitive model equates
this with the failure of he system
a. this model believes that unreported
crime is a result of social policies,
not the system
i. when a victim choses not to
participate in the system and
to partake beyond the
process, there must be
significant deference given to
the individual and wish to
deal with victimization in
their own term sand bad if
coerced or if they come to the
belief that there is no help
out there (learned
helplessness)
3. this model shares the same skepticism that
the DP holds
a. downplays the importance of crime
control strategies and that crime
prevention is heavily dependent on
social development in society and
the state in identifying areas in need
for individuals
b. doesn’t concern itself with blaming
the accused and once the crime is
14

committed, the aim of the circle is to


reduce harm to the individuals
i. really puts control into
victims’ hands as to how they
wan tot deal with
victimization and how the
victim controls the process
and if the process continues
ii. at any time, the victim can
stop participating in the
circle and if they want to
continue to proceed (the
choice of the victim)
c. victim has power to accept apologies
and reparations
i. victim punitive model, only
reparation is jail, probation,
fines etc. and these
reparations are made to the
state, not the victim and
allows for healing to be
individual in nature to the
individual needs and wants
ii. focus on factual guilty, but
only on why the criminal act
was committed
d. model doesn’t always accord with
what DP believes in
i. only way it differs from DP =
restorative justice
marginalized DP and makes
individuals take
responsibility
ii. DP and how the individual is
not forced to take
responsibility or guilt
iii. Must be what is best and
voluntary for all parties
6. Dominance of Crime Control before the Charter
a. The argument can be made that when you look at the criminal process in Canada
prior to 82’ we were in a CC model system a
15

 One could also argue that the DP protections afforded to an individual


charged with a criminal offence doesn’t come from the courts, but from
Parliament (in line with DP and Parliament being supreme)
- Parliament’s abolishment of capital punishment
 Before 82’, the truth was paramount and traditional discovery of truth was
the focus and not on the treatment of the person
- R v Wray
a. This case involved an individual being held in custody and
subject to a 10hr interrogation and wasn’t allowed to
consult a lawyer, this resulted in a coerced confession for a
confession of a gun used for a homicide. They believed
there was nothing wrong with it and no mechanism for
them to get rid of the evidence and the SCC not concerned
with this prior to Charter and of what happened outside of
the courtroom (how the police acted, and boundaries
violated)
i. This was in an era of broad discretionary powers
and how the police were given permission to
essentially enter to search someone’s house without
judicial authorization (writs of assistance)
 Before 82’, no right to bail and didn’t have the right to be given disclosure
for evidence against you and no right to lawyer or to consult one
 Prior to enactment of Charter, Parliament enacted protections for
individuals
- i.e. the Canadian Bill of Rights
a. rights were defined in minimal fashion and protection
under this didn’t supersede Criminal Law
b. If you look at the charter, is apart of our constitution and
the Constitution is the dominant law and what the bill of
rights was it was on par with the criminal code and didn’t
have greater power
c. Criminal code was not subjected to protections under
Canadian bill of rights
d. This approach changed significantly with enactment of the
charter and process was rearranged after the enactment of
the charter as apart of our constitution
i. Introduction of the Charter represents how these
rights are protected via checks and balances
Lecture Two: The Police
1. The Charter of Rights and Freedoms
a. Section 8 – Search and Seizure
16

i. What it guarantees  right against unreasonable search and seizure as


outlined by the SCC in a straightforward framework
ii. Framework:
1. An accused must demonstrate to the court that there was an
unreasonable expectation where they were searched and that it
unreasonably infringed on their Charter rights and was not done in
accordance to the basic principles of fundamental justice
a. Warrantless searches are presumed to be unreasonable
2. There is no section in the Charter which outlines what defines what
an unreasonable search and or seizure is and is defined by the
courts
a. There are 2 definitional aspects
i. Search  any state interference that interferes with
a person’s reasonable expectation of privacy
ii. Seizure  the taking of a things from a person by a
public authority without the person’s consent
3. The protection applies to people and not to a defined place 
looks at the issue of a person’s privacy (bodily function) and
informational privacy (what’s on someone’s phone)
iii. How to know if a person’s s.8 Charter right has been violated?
1. Has there been a search or seizure?
a. Not everything taken equate to a search or seizure but about
a reasonable expectation of privacy
2. If responded in the positive, then the government encroachment on
the reasonable expectation of privacy, then it is presumed to be
unreasonable
iv. The level of jurisprudence that a search and seizure is applicable in 3
circumstances (Crown must show these 3 elements)
1. Authorized by law  via provisions, legislation, or common law
that has authorized this search and seizure to take place
2. If the law itself is reasonable
3. And lastly, the manner in which the search in carried out is
reasonable, and have all 3 aspects, then it is reasonable.
a. In practical sphere of this, usually only focusing on the
first aspect, if its already been authorized by law
i. It is highly contextual and determined on a case by
case basis and not a one-size fits all answer;
generally something that the judge has to consider
as laid out in common law

b. Section 9 – Arbitrary Arrest/Detention


i. Guarantee
1. The recognition of the general principle that a person’s liberty is
not curtailed expecting accordance to the fundamental principles
17

of justice. Essentially, the court recognizes that your liberty when


resisted by the state is so highly valuable that it can only be
curtailed in certain circumstances when in accordance to the
principles of fundamental justice
a. Very basic set of protections made available to individuals
in Canada, but is not specific
ii. Need to determine two factors
1. Was the person detained (can be as simple as a cop arresting
someone or can be non-physical detention)
2. If the detention was arbitrary
iii. Definitions
1. Detention  requirement of some form of physical or
psychological form of restraint and is a suspension of the
individual’s liberty interests by a significant or psychological
restraint and can be as simple as an officer coming up and
grabbing your arm and asking you to “stay here”
a. S.495 of the CC outlines the powers of arrest that police
officers possess, essentially police must have reasonable
grounds to arrest you
2. Psychological detention  an example of this would be a cop
going up to someone and telling someone to remain put and the
individual believing that they can’t leave
a. R v. Grant
i. This was the seminal case in Canada which
addressed how detention encompasses both
physical and psychological forms and its
enforcement mechanism is found under s.24(2) of
the CC
ii. The case asserted that factors which would indicate
someone was being held under psychological
detention would be addressed on a case-by-case
basis. They held that the circumstances which arise
when being detained and how it may be reasonably
perceived by and individual when being
approached by a cop for the purpose of aiding
1. Is the officer giving you an order at an event
of a high traffic area
2. Is the cop investigating a criminal activity
and asking for witnesses
a. Its clear in these circumstances that
these is no obligation for you to stay
at the scene or location
iii. Factors
18

1. Is that cop approaching the individual from


what they suspect the individual and
involvement
2. The language used “sir/ ma’am, please stay
here”; where the interaction occurs and if
its in the presence of others
3. Characteristics of the individual (the person
who may feel like they’re being detained;
age, physical stature, minority status, or
level of sophistication)
3. Unlawful detention elements
a. There has been a detention (psychological or physical)
b. If the person being detained has been authorized, or if it is
arbitrary
i. Detention under s.9 is arbitrary when it is not
authorized by law and there are 2 main relevant
potential authorities for detaining an individual or
sources of reasonable authorized detention
1. May be authorized by statute (example 
when you’re driving, police have the
statutory authority to stop an individual on
the road at random for the purpose of
checking sobriety (authority defined under
HTA to detain and control your movements
and specific sections regulating)
2. the CCC has a specific section for arrest
and detention and gives the police the
power to arrest individuals without a
warrant under s.495)
c. Problem  is that detention and forms of restraint can less
drastic than arrest. The issue is that detention does not
equate with an arrest
i. The standard is that officers have to have probable
grounds to arrest an individual, the issue with
detention in the Charter is that it isn’t really
defined and the power to arrest is set out in the CC
and standard is there… then what is the standard
required to detain someone (this is left up to the CC
to determine)
ii. The interesting part is that s.9 which aims to set out
the scope of police powers when dealing with
individuals leads to the creation of new police
investigatory powers and new powers for
19

detainment prior to arrest (they define what


grounds and officer must have to detain someone)
4. Standards
a. Arrest  reasonable and probable grounds
b. Detention  reasonable suspicion
i. If an officer has reasonable suspicion that an
individual is involved in specific criminal activity,
they can be detained. However, when it was
intended to set out and limit the misuse that can
result from the State to citizens led to the creation
of new and more efficient ways to detain and arrest
people.
1. This fits into how due process is for crime
control (how s.9 created new police powers
that the police didn’t have before)
c. Section 10
i. Section 10(a)  right to be informed on arrest of detention
1. An individual must be informed promptly for the reasons for their
arrest
a. This means that if police arrest you pursuant to section 495
of the CC or if detained under reasonable suspicion or if
authorized by law, they must tell you why they’re arresting
you… cannot be a secret guessing game
b. Protection founded in a double rationale
i. Notion that no one is obliged to submit an arrest
without knowing the reasons for the arrest. The
officer must tell you why you’re being arrested, and
not obliged to go along unless the officer tells you
why you’re being arrested to know if to submit to
exercise of State power.
ii. Recognition that an individual needs to understand
for their detention and criminal jeopardy that they
may be facing. This is important because 10(a) tells
us directly relating to s.10(b) and a person can only
meaningfully instruct their lawyer if they know
what they’re under arrest for.
ii. Section 10(b)  right to counsel
1. This section outlines an individual’s right to counsel
a. Once you’re arrested, an officer must tell you why you’re
being arrested under s.10(a) and under s.10(b)
i. there are 3 requirements that officers have that the
courts have
1. the informational component
20

a. information is used within this


section and police must tell you that
you have the right to contact a
lawyer
b. Miranda warnings and right to
retain and instruct counsel which the
police have to tell you once you’re
arrested. This is a positive action on
the part of the State actor, and they
cant hide it from you
2. The implementational component
a. Police must give you the opportunity
to instruct counsel if you choose to
i. “you have the right to call a
lawyer”, and if you say
“yes”, the police have a
positive obligation to get you
in touch with a lawyer
3. Holding off
a. This means that once the police
inform you of your rights, between
that tine and time to speak to lawyer,
they have an obligation to hold off in
trying to elicit any other evidence
from you
i. Until you have been given
the reasonable opportunity to
speak to a lawyer, they can’t
keep trying to question you
and have the obligation to
hold off until vis a vis the
individual has contacted a
lawyer
d. Section 24(2) – Exclusion of Evidence
i. The Charter not only defines what a person’s interests are, what the state
isn’t allowed to do, but it also creates a mechanism where if the state
doesn’t protect your interests, it is held accountable; s.24(2) deals with
evidence
ii. Background
1. In the CC model, if the police barged into a house and found
quantity of substances in your residence, there previously was no
mechanism for if evidence should be reliable and prior to 1982,
more often than not, it would have been included
2. 24(2) deals with Charter violations when evidence has been found
21

a. For example, in,9,10(b) where there’s an unlawful search


or seizure, you turn to s.24(2) for enforcement. If arbitrarily
detained and evidence surfaces, turn to 24(2), if police
don’t honor Charter protections, then turn to 24(2) for
dealing with Charter protected interests.
iii. Aim of provision
1. The section sets out that if materials are collected in violation of
the sections included within the Charter the evidence shall be
excluded, as if included the evidence, the administration of justice
would be thrown into disrepute
2. Purpose  to maintain the good reputation of the administration of
justice and what it does. If the overall reputation of justice and
system is viewed in the long term and is adversely affected via
administration of evidence and the inquiry that a judge takes under
s.24(2), if information should be included or excluded is an
objective one and sets out that if a reasonable person informed of
all reasonable and reliable values in to the Charter would conclude
that the admission of evidence would bring the administration of
justice into disrepute.
iv. R v. Grant  how the court has developed analysis if info should be
included or excluded in terms of if the information has been discovered in
terms of a Charter breach. There are 3 branches that the court must look
at and weigh to see if the system of justice would be placed into disrepute
1. Seriousness of the Charter infringing state contact
a. How bad was the violation of a person’s Charter protected
interests
2. The impact of the breach on to the Charter protection of the
accused and how much did it undermine the protection of the right
of the rights of the infringed
3. The societal interests in an adjudication and dealing with a case
based off of merits and truth-seeking methods of process to be best
if sought via exclusion of evidence
a. The judge then weights all of these branches and asks if the
evidence is placing the administration of justice into
disrepute. And if “yes”, then it will be excluded; and if
“no”, then it will be used against the individual
2. The Charter Application Process
a. Background
i. Process prior to 82’
1. the system and application procedure was pretty simple, if a
person was arrested under reasonable and probable grounds at
the end of the day, matter goes to trial and Crown must prove the
actus reus and mens rea beyond a reasonable doubt. Prior, these
violations were dealt with outside the criminal trial process and
22

evidence not properly attained was not considered in the process;


only element that mattered was that it was proven beyond a
reasonable doubt
ii. system post 82’
1. in 1982, the Charter and created enforcement mechanisms. Due to
the creation of the Charter and process becomes more
complicated because what the Charter does is that it brings it into
consideration if a violation occurs and is brought into the process
and what to do and approach is apart of the process.
a. Unlike the CC model which was the dominant model pre
82’ the model most dominant post 82’ was DP
iii. 2 step process to prove that your rights have been unlawfully violated
1. Prove to the court that the police or state actor has violated one of
your Charter protected interests
a. If you can prove this to the court the i.e. the police entered
your house with no prior judicial authorization nor judicial
authority and not authorized by law, you’ve passed the first
hurdle because the police have discovered a gun in a way
that has violated your interests and breach of civil liberty;
if violated, then a judge will decide if information should
either be included or excluded
2. This goes to 242(2), what should happen to that gun in the
criminal trial process and if the Crown can reply upon that
evidence while proving reasonable doubt and without it, it cannot
prove the case and often in exclusion of evidence results in the
crown not being able to prove their case and mechanism that was
created to deal with state misconduct is subside within the process
b. Charter application example from CuLearn
i. DUI and breath sample administers
1. With the taking of a breath sample, they need to have a reasonable
demand to see if your BA is above a certain threshold; this can
give cops a reasonable threshold and right to detain and compel
you to give a sample in a BA machine
a. Being stopped in a vehicle is a form of detention
b. Forced to provide a sample on the notion of fearing
detention
c. Forced to give another sample
d. Sections 8,9, 10(a)(b) are engaged
i. you are giving up your bodily function to the state
and if police don’t do the procedure properly, it can
be apart of a Charter application to exclude
evidence and search being unreasonable and
detention being unlawful and if counsel rights
haven’t been properly applied
23

ii. Pre and Post 82’


1. Pre  the courts weren’t concerned with how they got it
2. Post  rights are enshrined and must show rights were respected;
if rights were not, that evidence should be excluded and because
its all apart of the process, you essentially have a trial within a
trial
a. If the Crown can’t rely upon them, then the Crown can’t
substantiate and prove their case beyond a reasonable
doubt and prove to the Court that the evidence will have a
negative effect on the justice system
i. The trial not just being built on the premise of
guilty beyond a reasonable doubt, but also on the
premise that how evidence was collected must be
considered in addition to additional constitutional
protections and enforcement methods
ii. The Charter application process resulted in
significant changes post Charter

3. Due Process Revolution and Retreat  this is what happens through the enactment of the
Charter
a. Rights to Counsel – Therens – Due Process Victory
i. RIDE – Revolution Retreat
1. Result of the Charter
a. Prior to 82’ the police enjoyed a crime control focused
state where the concern was investigating and prosecuting
offences by any means necessary and that watershed
moment happened in 82’ because of the enactment of the
Charter and now SCC willing to expand what these rights
meant to individuals and telling the courts how to use this
mechanism. What was discussed first under s.10(b), and
one of the first shots that happened in the criminal process
post enactment of the Charter and Therens
2. Right to counsel and Therens
a. R v. Therens was a case that was heard a week after the
enactment of the Charter. Therens, the accused, drove a
car into a tree and the police, who has reasonable
suspicion, demanded a breath sample at the roadside.
Therens cooperated and followed their directions and at no
point did he ask for a lawyer and nor did the police inform
him of his right to instruct counsel or the right to speak to
one
i. Example of how the police apprehended him on the
basis of factual guilt under the CC model and how
this turned into legal guilt quickly which ensured a
24

breathalyser to prove that he was not inebriated


and assumption that he’d plead guilty. In reality, he
plead not guilty and as a result his lawyers brought
a Charter application because the police didn’t
inform him of his rights to counsel under 10(b) and
didn’t comply with informational requests and
holding off guidelines. It was one of the first cases
to deal with informational requests
ii. Result of the Charter application was that the judge
held that the breath samples were to be excluded
and thus the Crown could no longer rely upon them
and the violation of his s.10(b) ought to be excluded
under s.242 and DP aspects prevailing over the CC
oriented set up
iii. “Factually guilty person going free” because of
this Charter violation
1. It was appealed up to the SCC and held that
it was an appropriate decision and the
refusal to limit and warning to instruct
counsel to those arrested and specific when
persons are detained, the state has an
obligation of the rights to counsel and SCC
concluded that the if the admission of breath
samples were allowed where the police
didn’t inform the guy to counsel, then that
right of counsel would be stripped of its
meaning and no earning under the Charter
legal rights
iv. Significant DP victory to those accused person and
held that if police don’t comply with the Charter
rights to counsel and additional evidence derived
that the evidence cannot be admissible. Therens
was limited to its effect and application because in
offence, you have to look at what you have after the
decision n is that the police implemented right to
counsel cards as a part of officer notebook and
right to counsel upon detention of arrest; and at the
end of the day, the procedure didn’t change much
besides ensuring the individual has right to counsel
1. Argument can be made that critique the DP
model and how the DP measures is for CC
for these circumstances because you have
decision where courts are concerned for
rights of accused but no one is concerned
25

about self-incrimination which is possible


because of self-incriminating breathalyzer
laws; on the flipside of having MAAD, an
individual who is obviously guilty is set free
because of a “technicality” and how this
technically is an important protection to all
individuals and portrayed that the courts
are preventing police from prosecuting and
would result in more “carnage” on our
roads
b. Result of Therens
i. We see a retreat of DP protections made available
to an individual
3. Ride
a. The Program
i. to reduce impaired driving everywhere by police
setting up random checkpoints where people are
stopped because they are driving is a reasonable
limit on a stretch of road. They then ask about if the
driver has had any alcohol throughout the duration
of the evening and ask for a breath sample if
reasonable suspicion is present.
ii. Individuals don’t have to answer to police and are
under no obligation to answer any questions in
terms of if you’d had anything to drink.
iii. Result  the police randomly stopping motorists;
involving the right to remain silent cannot be used
against you
b. Arbitrary detention
i. The detention is arbitrary because there is no
reasonable suspicion and one can argue that the
HTA gives cops statutory authority to stop your car
at random
c. RIDE issue
i. Once the police have detained you, the police have
the obligation to tell you why you’re being detained
and must allow you to call a lawyer. This
checkpoint is in violation of s.10(b)
ii. This issue went all the way up the SCC, they upheld
the ability for police to conduct these investigation
as a way to investigate drinking and driving were in
direct violation of one’s s.10(b) right, however was
upheld under s.1 (reasonable limits); this meant
that the courts said that while the RIDE check
26

points were contrary to one’s s.10(b) Charter


rights, it was demonstratively justified under 3
criterions to investigate:
1. Drinking and driving offences
2. Offences and detention is limited
3. Deterrence
d. Criminalization of politics
i. Dealing with drinking and driving accidents and
deaths by creating laws to detect and punish those
caught and not looking to educational programs or
intervention programs; instead relying on the
criminal sanction to deal with these behaviors
e. Result
i. 10(b) issue and checkpoints held that they were not
justifiable for longer periods of detention, “what
happens if someone is stopped with reasonable
suspicion and officer deploys another officer has to
bring person to unit and person there is under
detention; courts have found in these cases that
there was no longer a short period of detention and
no longer someone about a ride checkpoint (r v.
Grant and checkpoint of 30 mins of waiting in
contrast to Mellenthin and how it was found that if
police abuse these powers of detention to
investigate other forms of criminality and in
violation of charter interests and ought to be
excluded).

ii. Police Interrogation – Due Process Protections


1. Protections
a. When police have arrested an individual and want to get
that individual to confess and results in them confessing, it
must be protection under sections 10(a) and 10(b)
b. R v. Clarkson
i. The female accused has called her sister that her
husband had been shot and found her husband with
bullet in head. She was later informed of her right
to instruct counsel and the police charged her with
the murder of her husband because the police had
overheard her saying inculpatory statements to her
aunt; her aunt kept insisting that she seek a lawyer
1. Application was brought up in trial and the
statements were found to be in violation of
27

s.10(b) and appeal overturned resulting in


her conviction
2. SCC and issue if police violated her rights
a. Found that evidence of confession
ought to be excluded for her and that
she ought to have been sober and in
a position where she could
understand the importance of the
right to counsel and where a
confession was excluded
b. Where someone who is factually
guilty and evidence to be excluded
c. Rise to holding off like in Mellenthin
where SCC defined holding off
period for police to adhere to if they
wanted to illicit info from an
individual. Even if you consult
counsel and lawyer, in most cases, a
lawyer will instruct an individual
and not to make statements, doesn’t
mean client wont say anything and
will withstand a police
interrogation.
3. 3rd component and informational
a. Right to counsel is good but is
worthless if this right to counsel cant
be exercised and what if person
accused can’t afford a lawyer
iii. Legal Aid, Holding off, Technical Issues
1. Legal aid
a. Regina v Bridges
i. In this case, the police investigate a suspect and
asked for free legal advice because no money for
lawyer; the officer then asked them if they were
trying to hide something and tried to dissuade them
from calling a lawyer. The accused then makes an
inculpatory statement that the Crown wants to use
then allows them to consult a lawyer.
1. Charter applications were then brought,
and case held that the police must hold off
on the interrogation until an individual can
consult a lawyer, also including that free
legal advice must be given to individuals
who cant afford a lawyer
28

2. Holding off
a. Cases like this allow the court to protect individuals with
little resistance by state or state actors
i. This case and its proceedings are in like with how
DP is for CC in the way that police have quickly
adapted to these said DP victories
ii. Therens and how the police must inform individuals
their right to counsel and simply made notebooks to
read verbatim to the accused
iii. Post-Bridges, Canada acted quickly to set up free-
duty counsel or legal aid numbers for those who
couldn’t afford a lawyer; this was provided via
notebook to consult counsel and 1-800 number
3. Technical issues
a. R v. Prosper
i. How the SCC didn’t require all provinces to
establish this number
ii. 2 key decisions as a result
1. If a province has it established, the police
must give it
2. Provinces were not bound or required to
establish these hot lines
a. The reality however is that all
provinces now have these hotlines
set up for duty counsel to give advice
to detainees
b. Why do you think these informational, and implementations
of holding off were established?
i. If in holding off, you’d be waiting forever
ii. Police and provinces found it advantageous to set
up these lines so that they could interrogate and try
to illicit information from the accused  what we
see from the right to counsel is the development of
the quasi-absolute exclusionary rule
iv. Quasi-Absolute Exclusionary Rule
1. What it refers to
a. The rule refers to counsel under 24(2) and where the state
has violated a person’s right to counsel under 10(b)— in
most cases, the evidence was excluded so in most cases the
right to counsel was derived from this and the right to
excluded and based on the premise that a conviction
resulting from a unfair trial would be a travesty to the
criminal justice system in Canada
29

i. Evidence that is subject to problems with reliability


and credibility of false confessions
ii. 24(2) analysis where evidence derived from
Charter breaches was excluded and what we’ve
seen overtime as well is that the courts have
somewhat retreated from the expansion of the right
to counsel and really narrowly defined right to
counsel under s.7 and how this also includes the
right to counsel
2. Can the State use silence against you
a. No, this is not indicative of guilt and person has the right
to say something whether in trial and follows an individual
throughout the criminal process, not just in a police
investigation, trial judge cannot infer guilt from silence
3. Retreat
a. What we’ve seen over time is the SCC retreat both under
s10(b) and under silence.
v. Retreat of 10(b) – The “iron” triangle
1. Series of 3 important SCC decisions of what the police can do in
an investigation and what is considered a Charter protected
interest; the courts have slowly narrowed the protections to
individuals and the narrowing of it in reference to protections in
the US in regard to rights, silence, and counsel
a. Oickle (2000)  creation of rules around if a statement an
individual has given has been voluntary
i. In this case, the interrogation the police were
conducting were over a series of fires and arson,
which took place over 6hrs in a police
interrogation.
ii. State must show that the statement was given
voluntarily and not under circumstances such as
under torture and how under this condition, may
have been given involuntarily (sleep deprivation
and the room being at 10 deg.)
iii. Situations where a statement may be involuntary
and where certain inducements may be brought (“if
you confess, we won’t press charges” etc. and if the
inducement is strong enough to produce a
confession, and if proven, can be excluded
1. It is a high barrier to get receive evidence
exclusion from a confession like this
iv. Decision  the majority of the SCC ruled that the
confession was voluntary despite them minimizing
the significance of crime. Only suggested that
30

mental help and fiancée would have to undergo


polygraph nut wouldn’t if they confessed and said
that it was indeed voluntary
b. Singh (2007)
i. He was arrested for the 2nd degree murder of a
bystander shot by a stray bullet and invoked his
right to silence 18-times in the interrogation
ii. While he never confessed to the commission of the
offence, he proved evidence that the police
corroborated; in making these inculpatory
statements, he had put himself at the scene of the
offence and said he knew the person
iii. Decision  the majority of the SCC decided that
the statements were voluntary and not that the
police were systematically breaking down his mind,
just because the person has involved their right to
silence, doesn’t mean that the police can stop
pursuing their custodial interrogation. Here, the
right to remain silent doesn’t mean that the police
will stop asking you questions and interrogating
you.
1. Compare this to the US where the
interrogation stops when an individual
invokes their right to remain silent right
a. The Reid technique and the
subsequent result of unreliable and
false confessions
c. St Claire
i. Decision  held that the right to counsel doesn’t
mandate the right to defense counsel via
interrogation
ii. How a person doesn’t have a 10(b), must consult
with a lawyer and how there is no further
obligation that the police are bound to; this means
that the police are not legally bound to offer more
than one consultation with a lawyer and have no
obligation to give you the opportunity to have a
lawyer present in the room during the interrogation
— the only aspect which they must comply with is
allowing you to consult legal counsel
1. Police can allow you to re-consult with your
lawyer if dealing with different legal
jeopardy than what already is being
interrogated for.
31

iii. Right to counsel  call to lawyer and them telling


you to keep your mouth shut thus watering down
these responsibilities and protections
1. From DP revolution and little by little, the
SCC narrowing down what s.10(b) really is
and means and the subsequent protections
that are allotted to individuals
b. Arrest and Detention – Crime Control Affirmed
i. Investigative Stops
1. Being pulled over while driving
a. Provincial legislation giving cops the ability to stop
motorists at any point for any rational reason connected to
the HTA (fitness of vehicle, licence check, and sobriety of
driver)
b. SCC post Therens and RIDE checkpoints
i. Saved under s.1 because the purpose of stopping
drinking and driving and thus benefitting the
greater good; harm prevented greater than harm
that could be caused (via managing risk of drinking
and driving to the community)
ii. Concept of risk used to curtail rights available to
individuals and very CC oriented in approach and
limitation of rights to individuals to try and deter
and detect criminal activity happening
c. Issues with dissent of stops (Ladouceur)
i. Case overview
1. the dissent in the case was concerned with
the vase powers vested with the cops, under
this provincial legislation allowed for a cop
to stop individuals at random on a whim
without any grounds whatsoever
a. under HTA (apprehension), cops
don’t need to merely believe that
you’re not registered to pull you
over
b. underlying concern  police power
will be misused and target visible
minority groups for stops and
investigations
i. media wasn’t too worried
about police having these
powers to stop and
investigate people based on
suspicion and not on conflict
32

between DP rights and CC


approach and no real victim
speaking to these expansive
police powers
ii. concern being that police
will rely too heavily on these
suspicions and hunches to
conduct criminal
investigations and use it
under the guise of an HTA
violation. Example  police
see someone and have
suspicion of what they’re
doing (i.e. drug trafficking
and use vehicle to make
deals), police, without
reasonable suspicion to
detain them can use the HTA
to stop them and gather and
further their investigation as
much as possible.
iii. HTA allows police to stop
people at random and how
this power is often misused to
try and further a criminal
investigation.
2. Arrests
a. S. 495 of the CC and allowing cops the grounds to arrest
people on probable grounds and creation of common law
powers for arrest, in addition to search and seizures
i. Development through common law and
jurisprudence; if a cop properly arrests someone,
they have the grounds via common law to search
the individual or vehicle that the individual is
driving— how a warrantless search is justified via
common law
1. Police arresting someone in their vehicle
now have power under common law to
search and to further their own
investigation; reasonable and probable
grounds standard is not met to perfection
b. Probable grounds and R v. Storey

ii. Bail and PTC


33

1. Today
a. Today, this is an expansive in nature today, found under
sections 515 and 495 of the CCC
i. 495  police having the power to arrest people
without a warrant. The CC under this section, the
police maintain discretion at the time of arrest to
determine if a person should be released or
detained; and if they should be released pending
the disposition of their charges
2. Ways police can release someone back into the community
a. s.496 Promise to appear, document having someone’s
first court date and person signing a document
i. lowest form of release
b. s.498  release from custody by officer in charge, the
officer in charge can decide whether to release a person
into the community with one or more conditions other than
them just promising to appear at the closest court date
i. number of powers allocated to police officers to
ensure person is released under appropriate
conditions
c. s.499.2  conditions of a peace officer can give to an
individual (i.e. to stay in Ontario, not to talk to certain
individuals etc.)
d. if peace officer determines they can’t be re-released, there
is a requirement for the individual arrested to be brought
before a presiding justice within 24hrs
i. at this point, a police officer determines they can
release them, then its up to the Crown to see if the
release can be met with consent, then judge, or
justice of peace can be released at a bail hearing
3. Bail  if someone can be released into the community pending
disposition of charges
a. 515.10 sets out 3 grounds where a judge can order
someone to be detained in custody until charges are dealt
with wither way via trial or plea bargain
i. Primary grounds  necessary to ensure persons
attendance in court (if a flight risk) and can see
whether this may arise if a person may be living in
a different jurisdiction where they may not have
connection to the community where the charges
arise in
ii. Secondary grounds  focused on the protection of
the public and complaints. This level sets out how
an individual can be detained in custody if there
34

lays a substantial likelihood, they will commit


another criminal offence or if released, will
interfere with the administration of justice (i.e.
going after a witness)
iii. Tertiary groundsthis is a catch-all ground where
the concern with the maintenance of the confidence
of the public in the administration of justice and
what ground it looks at and if the individual is
released and how a member of public would look at
this. The CC outlines:
1. The strength of the Crown’s case
2. Gravity of the offence
3. Circumstances surrounding the commission
of the offence and if found guilt, would a
mandatory minimum be in effect
a. This ground should be left for the
more serious criminal offences
b. Secondary and tertiary grounds have developed over the
years and have changed
i. Sub b brought in that any individual under the age
of 18 years (child porn and physical sex offences
against the young)
ii. Sub c, 3 and 4— language about firearms and
newly added by the conservative government to
deal with the public concern with increasing gun
offences (Year of the Gun in Toronto and in
Ottawa, the shooting of innocent bystanders, boxing
day shoot out by rival gangs
iii. Language reflecting to give judges and justices of
peace to better detain individuals who have
committed these offences
c. Language in bail
i. Concern about crime control and about controlling
risk of future offences and of offences that go to the
heart of public safety including firearms and crimes
against minors
d. Changing landscapes
i. Despite the protections listed under s,9 and the
right to bail under the Charter, what we’ve see
more and more people being detained in custody
and more people being denied bail and are placed
in detention facilities
1. More people in custody that haven’t been
found guilty of a CC offence
35

a. Blame 515.10 in regard to how


people are concerned about risk
which is how this is a determinant in
bail
b. Being held in custody instead of
being free until proven guilty
2. Cases
a. The St-Cloud (2015) decision and
this expanding territorial grounds
for the courts
b. Queen and Antic (IMP)
i. How the process of denying
bail is out of control
ii. How we are becoming a risk
adverse society, and how our
criminal justice system is
reflecting this (i.e. the police
are no longer releasing
people, and neither is the
Crown)
iii. The Crown not releasing
people out of custody
because they are scared of
the risk which may result
from their release and have
resulted of releases
occurring within tight
conditions (i.e. house arrests
and surety supervision)
iv. We think we see a DP
revolution with bail under
the Charter, how s.515
displays the retreat of the
courts detaining individuals
and more revolution in Antic
in 2017 which the court
focusing on the presumption
of innocence and that the
right to bail is important and
detention should be the
exception, not the norm and
not fall a foul to s. 10(a)(b)
(c)
c. Queen and Myers
36

i. The SCC tried to focus the


analysis back to the basic
presumption of innocence,
right to reasonable bail, and
that detention should be
exception and not the norm.
c. Search and Seizure – A smarter Crime Control
i. Writs and response
1. Writs of assistance  allowing entrance into places without prior
judicial authorization and prerogative power was challenged and
SCC struck down powers given to authorities and found
unconstitutional and they authorized searched without prior
judicial authorization
2. Unconstitutional searches
a. Searches that are unreasonable and unconstitutional are
violations of s.10 Charter right and once you show its
unreasonable, the presumption can be rebutted
3. Parliaments response
a. Parliaments response to the striking down of writs of
assistance was that the circumstances where changing and
fast paced situation and want to get a hold of evidence and
writs of assistance used to help them do this
b. They created a tele-warrant system under s.487.11
i. This allowed police officers to apply for a warrant
via telephone instead of going up in front of a
judge, outlying their reasonable and probable
grounds
ii. Now police officers can call into a phone number
ad talk to a judicial officer and fax officer to justice
of peace
iii. Writs of assistance go down and enactment of tele-
warrant system  appeal of writs caused
parliament to enact more efficient ways to warry
out a search warrant
1. DP revolution with the SCC striking down
writs of assistance which result in a CC
response from parliament and procedure
created that search warrants are granted as
it is presumptive reasonable and not up to
the accused to prove it was presumptive
unreasonable
ii. Impaired cases and blood seizures
1. Early cases
37

a. Early cases of exclusion of blood samples which would


have been afforded evidence from the police obtaining
blood samples without prior judicial authorization s
i. Important serious nature of taking blood from an
individual (intrusive form of search against an
individual causing a person to give evidence
against themselves via blood)
b. Parliament’s response  blood warrant
i. What parliament does here by creating this warrant
is giving the police the ability to request via i.e.
tele-warrant a request to obtain a bodily sample
(blood) from an unconscious driver where accident
caused death or bodily harm
c. What happens  DP revolution of court striking down the
unreasonableness of obtaining blood without authorization
and parliament’s reaction by allowing police to get your
blood and can do so at any hour of the day because of the
tele-warrant system
iii. Warrantless Search and Response
1. What this includes
a. Wiretaps, trackers, etc.
i. All these forms of investigation and investigative
techniques were struck by the SCC
ii. A search that is warrantless that involved intruding
on a person’s privacy = unconstitutional and not
done in accordance to the principles of
fundamental justice; if you keep using them without
judicial authorization, you can’t use it against the
individual
2. Response
a. Parliament’s response was the creation of legislation
which increased police effectiveness
i. Police general warrants (police obtaining this for
investigatory techniques involving a search and
getting prior judicial authorization approving this
behavior and warrants to deal specifically with this
— i.e. wiretapping and searches
1. The CC outlines that for wiretapping, you
can see how the SCC invalidated
warrantless searched which lead to a
retreat via Parliament through the creation
of new expansive police powers and
techniques which engaged in prior to
38

Charter = unreasonable because of prior


authorization
2. The creation of authorization and process
that officers can use to get ahead of time
a. When office gets a warrant to seize
something, as defense counsel, it is
hard to get that invalidated
3. ONCE THE AUTHORIZATION IS GIVEN,
THE SEARCH IS PRESUMED LEGAL
iv. Not-so-absolute exclusionary rule
1. In contrast to the quasi-absolute exclusionary rule used to enforce
the expanded right against self incrimination, the SCC was
reluctant to exclude evidence obtained from a simple search and
seizure
2. Most of the time, evidence obtained through search and seizure
violations would only be excluded if the seriousness of the
violation outweighed the seriousness of the charge
3. R.v Collins
4. R. v Grant
a. The court interpreted the right against unreasonable
search and seizures in a generous DP manner, but did not
pay the CC price of frequently excluding illegally seized
evidence
5. Searches which have warrants make the courts less included to
exclude real evidence from a search or seizure and its hard to
prove that evidence obtained with a warrant to be excluded,
especially with a warrant, and now, even without one…
v. DNA
1. Use of DNA is a highly intrusive manner of evidence collection
without prior judicial authorization
a. Protective of accused persons where the information was
given to the police without full knowledge of what the
evidence would be used for
2. R v Borden
a. The DNA sample taken from the accused (who didn’t know
what it’d be sued for) was used for the homicide
investigation for the DNA found at the scene
b. Decision  SCC found that the accused’s consent was
obtained improperly because police didn’t fully inform him
of what the sample would be used for
i. This case creased jurisprudence for consent
searches for the police being required to receive
consent to seize and search appropriately
39

ii. Court found that a person must have full knowledge


of what’s being taken and how, why, and for what
its being taken for— can be used for possible
evidence jeopardy (until then, consent is given
which then indicates that it is an appropriate
measure)
c. Parliament then invalidating warrantless taking,
responding with sections of the CC for samples and bodily
impressions (teeth marks, fingerprints etc.)
i. This authorized police to apply for a warrant to get
a sample to seize samples from individuals
ii. Requirement for authorization  must be given by
a judge a head of time and having reasonable and
probable grounds that the sample would provide
them evidence for the commission of the criminal
offence, and on that basis, Parliament created
provisions for this authorization and searches
iii. Apart of giving authorization for searches and
seizures, this relies upon victim’s rights as
justification for creation and tools for the police
1. In order to protect victims and prosecutor’s
cases, we need that legislation and police
used justice expansion of legislation and
victim’s rights and enactments of popular
legislation
d. Feeny  search and seizure
i. This case dealt with search and seizure
ii. Decision  the SCC reverse a rule which allowed cops to enter homes
without arrest because under the Charter, privacy interests of an
individual in how it outweighed the interests of the police
iii. Prior to Feeny
1. Police could enter one’s home without a search warrant
iv. Feeny
1. The police entered his home to find a bloody shirt, this was
excluded because the SCC found that by entering someone’s home
without prior judicial authorization was unconstitutional
a. Parliament responded with authorization to police for
warrants to enter someone’s home in order to arrest them
b. Warrantless search executions were presumed
unreasonable (Parliament responding with authorization
procedures for police to enter homes in this manner)
c. Parliament responds with entry and arrest warrants
i. Even to this day, we call these warrants, “Feeny
warrants”
40

ii. Legislative authorization for search of a person’s


home if exigent circumstances of real possibility of
evidence destruction
4. Due Process for Crime Control?
a. S. 10(b) and s.9 actually represents how DP is for CC and does it by leading a CC
oriented approach
b. Pattern emerging
i. DP decisions of holding off and unreasonable actions by police and all
decisions greeted by short orders in Parliamentary authorizing same
search, but in different ways in accordance to the Charter
ii. Decisions of SCC provoked legislation from Parliament that gave the
police more expansive powers in line with protections afforded to
individuals under the Charter
iii. Search and seizure = strongest support for how DP was ultimately for CC
Lecture Three: The Criminal Trial
1. Charter Application
a. Section 11(b)  right to be tried within a reasonable amount of time (not a
protection that deals with police powers, but more of a procedural protection
which is available to individuals and Charter protection to get a trial within a
reasonable amount of time to get someone from factual to legal guilt within a
reasonable amount of time
i. What is a “reasonable amount of time”?  nothing defined in the CCC,
and is left up to the SCC and Courts to decide – Parliament didn’t define
this, it was the SCC which had to determine its applicability and what it
meant
b. Section 7  disclosure (life, liberty, and security of persons and not be deprived
unless in accordance to principles of fundamental justice and how this impacts
disclosure)
i. Disclosure  when police gather evidence in an investigation and refers
to the providing of evidence found pertaining to the accused that is not
subject to confidentiality issues
 this is not dealing with actual evidence, but procedural protections
if the right to a trial within a reasonable amount of time and what
if not provided with disclosure (24.1)
c. Section 24(1)  look here for enforcement mechanisms and is fairly broad; what
this means is in reference to procedural protections
2. Right to have a trial within a reasonable time 11(b)
 After 11(b) was entrenched in Constitution, there were some attempts via
Parliament legislating what “Reasonable time” meant and SCC deciding
what it meant; we also see the recognition of reasonable time via courts in
relation to trial within a reasonable amount of time
a. Askov  first case dealing with the issue of trial within a reasonable time
i. Background
41

 Askov was charged with CC offences which included extortion and


matters relating to evidence
 Was denied bail for 6 months, after he was granted bail
 Because of the nature of the offence, he was granted a preliminary
hearing (because its an indictable offence) and a 10mo delay
proceeded here between the arrest and laying of charges
 They attempted to get the quickest trial date, nearest date at that
time was 13mo away from committee and standing trial now
almost 2 years between arrest and trial
a. Because of lack of judges and crowns etc. to deal with
workload, the case didn’t proceed at the date and was then
off set to almost 3 years between the arrest and start of the
trial
b. At the beginning of the trial, his lawyer brought a Charter
Challenge application
 This was dealt within the criminal justice process
(similar to prior guidelines around police
misconduct)
c. This was unprecedented and new concept that procedural
protection and enforcement mechanisms under 24.1 and
part of the remedy was for the Court to stay proceedings
(court stops prosecution from continuing and charges
being dropped)  saying that the Crown has violate the
accused’s trial
 Result = 2 step procedure
a. Trial within a reasonable time is remedied
b. And why a stay of proceedings would be appropriate
 Important  because in recognition where he was being tried, the
judges were in struggle of more court rooms being built and
judges etc. and concern that caseload was exceeding what they
could handle
a. Judges  lobbied government and resources at the time to
not increase money to justice system and problem
continuing to grow greater of backlog of cases from case to
trial
 An aspect that judges mentioned in 11b analysis is
that if needs for the court system is not met and if
government does not provide adequate funding,
they will have to deal with angry victims
 Case was then appealed to the Ontario court of appeal and they
reverse decision and follow a CC oriented approach reaching
conclusion that stay was unwarranted
42

a. Assumed that Askov was factually guilty and used it as a


sword instead of a shield
b. SCC reversed decision and stressed that all persons
accused of committing an offence were innocent until
proven guilty and the accused should be provided the
earliest available opportunity to defend themselves
 SCC  granted a stay of proceedings and reversal of stay
a. SCC looks at the waiting for trial as agony of the innocent
person waiting to clear their nae
b. Ontario Court of Appeal’s perspective was that it was
instead a loophole which would be used by the guilty
c. Result  24mo = unreasonable
 Only remedy pretty much was a stay of proceedings
and creation of a very quasi-absolute rule seen in
counsel cases where if shown a violation of a 10a
or 10b rule for evidence to be excluded and if rights
were violated = stay of proceedings would ensue
ii. Fall-out
 SCC in Askov asserted that 2-8 months was a reasonable time
frame for an individual to go from arrest to trial
a. Problem  lower courts and prosecution followed the 6-8
mo. guideline literally, while Askov never said that if
falling outside of the range would result in a stay of
proceedings – lower courts and prosecution viewed it as a
limitation period instead of as a guideline
iii. After- Askov
 Result of introduction of guidelines
a. Increase in stay of proceedings granted by the courts as
they pertain to criminal charges; said that more Crowns
and Courts must be provided
 Prediction of Courts
a. The courts predicted the public outrage which would be in
response to the government’s actions and lack of funding
 Courts knew the government would get blamed for
not funding the system appropriately, however, in
reality, the government utilizes the media to their
advantage and creates through the media an
impression that the courts are creating amnesty
programs for criminals and flipped the script
1. Government persuaded the media and said
the chaos was the courts fault and blamed
them
 Public outrage
43

a. Public outrage as a result of government’s media story in


re: to the decision of the Courts in Askov, not with the
government and how they handled the underfunding of
judicial resources
 How the courts combatted the outrage
a. They released stats of how many cases were stayed and
how the purpose of Askov was overinflated by the
government and how the government used and portrayed
the stats in different ways than they should have
 Government’s report
a. They reported charges being stayed, not cases
 In a case, you can have someone facing multiple
charges before the courts and how the governments
stats were in relation to specific charges and not to
the cases
1. Government was reporting on all charges
and also reporting on all charges which
were stays in proceedings  this left the
impression that al of a sudden, there is a
tremendous increase of charges being
stayed
a. This was not necessarily the case for
Askov, but also could have occurred
for different reasons and said that
because of that number, it caused
people to run free – media failing to
report that 90% of charges were
stayed because of Askov when in
reality, only 10% were stayed
because of Askov
 Example  you can have a case with a BA over 80
and driving while impaired. In this case, you can be
charged with both in context of one case; principle
in criminal law that you can be convicted of
offences similar to one another if occurring similar
to one-another (if convicted of the impaired, can be
convicted of both in one— the Kiennapple
principle)
b. Court in Askov warned Crown and lower courts to not take
a rigid interpretation of guidelines set
 Reality  leading to increase of stays of charges
and public outrage
 The sensationalizing of statistics and construing it
in a way that says that the courts didn’t care for
44

victims; worry from defense counsel is that this


decision would evoke a CC response and increase
in sympathy for victims
 What we see post Askov
a. Government increasing funding
b. Amount of charges disposed of increase (charges dealt
with outside of the stay of proceedings = more efficient)
not much of a decline post Askov and prison population
was actually growing shortly after
 i.e. search and seizure cases where a CC response
was provoked and resulted in more charges being
disposed and more people moving from factual
guilt to legal quicker and quicker

b. Morin – Retreat?
i. Morin is an example of the SCC retreating from its own decision in Askov.
Until this case, the SCC was unable to revisit Askov, and what resulted in
it
ii. the case was premised on drinking and driving charges where the trial
court stayed proceedings outside of 6-8-month guideline. It acknowledged
the public reaction to Askov was mixed and not received well (public
perception to Askov was amnesty to criminals being granted)
 justice in Askov, victims conclude that the court system cant
perform its most basic functions; victims rights creeping into
analysis
iii. this case recognised the importance of crime control and its role in the
justice system crime control in the justice system and the importance of
victims rights and needs and the important to the right of a speedy trial
 The Supreme Court of Canada did not overrule Askov guidelines
in stressed not to be applied in a mechanical fashion and needs to
be flexible and knowledge like the limitation period.
 One of the things some increased forms 6-8-month form 8 to 12
months for low level stuff assault and impaired driving
iv. what they did was overturns day preceding wish granted the lower court
13-month delay was reasonable in trying to undermine be flexible analysis
of be 812 months guideline and show 13-month delay and how it was
reasonable under the circumstances and recognise public involvement of
the trial
v. bring speak use actions into focus and behaviour of the person through
his or her lawyer is impeded through the delay and can't purposely expect
to stay of proceedings given
vi. brings in considerations of prejudice given to the accused and not looking
at the delay caused in it's cause but also including report from the court
through prejudice
45

 must show prejudice showed as a result in addition to stay long


enough
vii. This case shows how due process is vulnerable to shifting judicial
attitudes that the courts not being committed to you when crime control
and victims’ rights becoming more publicised
viii. Jordan sets out limitation period and does the opposite of what the court
was trying to do in Askov and Morin and sets out frameworks
c. Frameworks
i. There is a presumptive ceiling and limitation period of when a trial is no
longer reasonable amount of time
 Charged with an offence and proceeded via summary conviction
and Ontario court of justice deals with it
 Charged with and indicatable offence, can have matters before the
Ontario court of justice of before a superior court
a. Crown can proceed via summary or indictment  we have
in Ontario, the Ontario court of justice which only deals
with summary matters and does preliminary hearings and
proceeds all within the same court
b. Via indictment, can go to trial in OCJ or can go to trial in
the superior court of justice
 This can be via judge or jury
ii. Presumptive ceiling
 If matter proceeding via OCJ, 18 mo. And after this it is
considered presumptive unreasonable
a. OSCJ matters = 30 mo. From start to finish
 You subtract the following from the time period 
any delay waived by the defense
 If doesn’t conclude within the guidelines =
presumptively unreasonable
1. Contrast this approach to Morin and Askov
a. 6-8 months
b. 13 months
i. Look at the increase from 6-8
months to 18 months of what
the courts consider
unreasonable
ii. If ceiling is exceeded, then it
is presumed to be
automatically unreasonable
 Similar to search and seizure investigations, you
can rebut the assumption under s.1 under the
following circumstances
46

1. A discrete event occurred which was


reasonably unforeseeable and unavoidable
such as i.e. if a witness is ill, event happens
at trial such as a fire alarm going off
a. These delays due to discrete events
subtract from the total delay
 This shows that the case is a complex one that the
Crown has to deal with
1. Either the nature of the evidence
2. Seriousness of the charges are not factored
in
a. Says that murder charges don’t fall
automatically under consideration
and must show nature of evidence or
issues being complex
b. Under this, the judge can take off
delay
i. If court cant get delay below
18 months, then a stay of
proceedings is granted
ii. If delay is under ceiling, then
the accused can still bring a
stay of proceedings
application, but the onus
placed on the accused of
something that is
presumptively reasonable is
actually unreasonable
 Presumptive unreasonableness
1. If the accused establishes both elements,
then charges can be stayed, and much
different in approach than what is decided
in Askov
a. Concerns with judicial resources
available and concern for the
judges, crown, and courtroom space,
and more mattes taking longer to get
to trial and government failing to
fund the judicial process adequately
enough
b. As a result, we see an increase in the
length of time brining matters to
fruition
47

 Fall out from Askov we didn’t see the same


amount of stays granted in Jordan and not same
numbers in Jordan
1. Jordan created transitional periods
a. What this did was allowing escape
routes for those cases within the
18mo. Time period and excused
them because they happened under a
prior regime
b. The court and crown went to lengths
of matters being stayed under
Jordan analysis and crown’s
prioritization trying to resolve
smaller cases
i. Judges being more active in
clearing cases from dockets
in trial in trying to lessen
impact of Jordan and Askov

d. Godin & Lahiry – further DP revolution and start of retreat


i. Godin
 The Supreme Court of Canada after Morin sets out how to
determine prejudice
 Says of inherent prejudices as delay increases in more
prejudice equals delay  “has a witness died” is evidence
not able now because of delay” “has accused person
specifically suffered because of delay”
 Case says you can use these examples but longer
periods of delay results in inherent prejudices in
that rely on inherent prejudice
 inherent prejudice
a. Mornin and stay proceedings and how we must look at
prejudice caused because of delays and show how were not
able to work with the charges i.e. because a witness has
died
b. Godin says that the prejudices can be inferred based on the
periods of delay and that the onus is on the Crown and
what the judge will consider if the stay proceedings is to
occur
ii. Lahiry
 Supreme Court in this decision in not to appeal the appeal court's
decision
48

 this case puts a greater onus on the accused to get their trial
expedited as soon as possible and what this does is reverses the
requirement for accused persons need to show that he or she had
always wanted to matter to go to trial is quickly as possible but
also that's a judge in this case narrowed what's considered an
institutional delay
 2016 and SCC revisiting issue of delay
iii. Jordan and Cody
 Supreme Court in Jordan takes law post Askov and throws it away
and says that analysis and determinant within a reasonable
amount of time breached had become confusing And different
jurisdictions looking at the way form one aspect and how to
calculate this and how to calculate that the court essentially says
that this is becoming a game and beyond what is a trial within a
reasonable amount of time
 Askov and 6-8 mo.
a. Morin says that Askov was a guideline and must need flex
 Jordan sets out limitation
3. Right to disclosure – s.7  Askov problem linked to how cases were proceeding through
the criminal justice system and how it was being handled and how the Crown’s office
were trying to plea bargain and how they were being screened and precedents given to
that
a. Pre-Stinchcombe
i. Discovery
 People were unable to plea bargain and this would gum-up system
tremendously and can have a perfectly funded system where
everything does to trial and rely on CC model for speediness and
reality that there aren’t enough judicial resources to go around
 CC and there are only a finite amount of resources available and
the court being interested in this and how plea-bargaining results
in lessening the cases of those going straight to trial
 One of the ways that it became an accepted practice was via the
right to disclosure that the right evidence was all relevant via
investigators not being privileged
 Disclosure forced the Crowns and judges to assemble cases ahead
of time to place defense counsel in a position where they could
intelligently make a deal for their client
a. This means that the right to disclosure forced the crown
and police to put together an air-tight case to the defense
in a way that there were no way out and best way to limit
and resolve damage  without this, the system would
suffer from backlogs and without the right to disclosure,
the system was inefficient and risky
49

b. Without this, the system would suffer from backlogs and


without right to disclosure, the system would be inefficient
and risky
 Disclosure would lead to increase of plea
bargaining happening and kept the CC assembly
line operating smoothly
 Right to disclosure = DP protection to accused
1. Creation of DP to accused resulted in
system more efficient and system being more
like an assembly line
2. Prior to enactment to the Charter,
disclosure was not required via courts and
the Crown didn’t have an obligation to give
evidence to the accused person
 Issue  wrongful convictions
a. Donald Marshall
 One of the main causes was the failure on the
behalf of the Crown to show inconsistencies in
evidence and thus the Crown wasn’t able to
adequately cross examine the witness
 No situation where Crown is required to disclose
evidence to the accused and if person should be
given the right to disclosure where they fulfill void
parliament left
b. Stinchcombe  seminal case dealing with disclosure
i. Background
 Stinchcombe was a lawyer accused of embezzlement and of the
witnesses was his former assistant
 His lawyer tried to interview which she refused and requested that
the Crown and police had over recordings of her evidence and
forcing it and the trial judge denying it
ii. SCC = warmer request
 This was used to show the principle that the Crown is required to
disclose all relevant and non-privileged information, where it
harms of helps one another and it must go to the Défense and
disclosure principles
a. The court justifies the placement of respect on the Crown
attorneys of wrongful conviction of Donald Marshal and
had the defense been given the evidence, Marshall may not
have been wrongfully convicted
 Right to accused having disclosure is the equivalent
o fright to full answers in defense and right that a
person has a right to fully defend themselves and
50

must have opportunity to respond and counter the


Crown’s case and court held that the recordings
were to be given to the accused
b. SCC notes that the requirement that the Crown has to give
disclosure early may actually lead to less trials and by
placing this requirements and maty have Crown taking and
withdrawing case and person through their lawyer may be
strong and resolve and limit damages as much as possible
c. DP decision like this and recognition of rights would
facilitate more of a CC model and no secret and warmly
backing by courts during Askov crisis as thus was a way to
ween out the system through deciding which cases should
and shouldn’t go to trial
c. After-Stinchcombe
i. What Stinchcombe meant to the criminal process
 DP oriented case where Charter protections constructed in a way
to full answer and defense led to a more CC response in our
criminal justice system
 Main thing it did outside of an accused having all relevant and
non-privileged disclosure, but it also legalized the right to prove
guilty
 Factual to legal guilt as soon as possible and person taking
responsibility for what was done, and person will agree that they
have created offence and proceed to sentencing without Crown
having to prove a case beyond a reasonable doubt
a. DP model holds belief that the criminal trial is the most
important in the process and that people should go to trial
and plea bargaining is bad and state should always prove
the case beyond a reasonable doubt
ii. Post-Stinchcombe and the response of various actors in the criminal
justice system
 Police officers
a. Police officers in investigations need to be more
throughout when gathering information, and information
gathered must be given to the Crown attorney
 Administration
a. Designed to increase amount of plea bargaining and to
essentialize it as a part of the carinal justice process
 Prior to enactment of the Charter, persons pleading
guilty = conviction of counsel
 Mentality of how could you reasonably prove
someone guilty without knowing the evidence
against you
51

1. This attitude reflected Packer’s assumption


that without a trial, the accused would not
and could not be treated fairly
a. As time passes on, pleading guilty is
recognized by actors within the
system and was regulated in essence
b. Acceptance for actors within the
court system to engage in resolution
to try and reach resolutions for the
person’s charges
i. Charter protections enacted
under legal rights enhanced
the work produced of police
investigations
ii. Meant that cases are more
throughout investigated in
how the Crown and cases are
becoming more air-tight
iii. Enhancement in quality
means that most accused
persons ended up pleasing
guilty
b. One of the task forces implemented = the Martin task force
 Acceptance for role in plea bargaining had in the
cjs
 Not bad thing and something to alleviate pressures
that the cjs had on an everyday basis
 Acceptance of how work at the start of a file via
police or crown in screening would save
considerable amounts of judicial resources
 Model of dealing with cases like in Peel region and
Crown’s not resolving matters and taking position
that guilty must plead to all
1. Not model and Crown and actors of cjs had
to change ways to at lead engage in these
types of discussions
 Guilty
a. Reduced sentences and plea-bargaining g
b. Mitigating factors to have plead without the State to have
to prove case beyond a reasonable doubt
c. Person has accepted guilty and sign of remorse
d. Save on judicia resources and person to trial from start to
end = save system considerable money and this is
mitigation on behalf of the accused
52

 Reaction especially in cases involving vulnerable


complainants like young people and children, to
have them from having to come to court and testify
and judges encouraged to go along with resolution
discussions
 Example: in JPT  joint positions where the
Crown and defense agree and is presented to judge
and is where Crown asks for appropriate sentence
and defense makes their submissions; they go back
and both and then the judge make their own
dismissing (i.e. open-sentencing positions)
1. Judges encouraged when considering a
joint recommendation to adhere to it and
not go above or beneath the
recommendations
2. Judges always keep discretion in these cases
and hearing to what they see fit
3. Only way judge would go above or below
the recommendations is if the sentence
would lead to the criminal justice system
being put into disrepute
4. Judges are encouraged to facilitate
resolution discussions between the Crown
and defense
 at the end of the day, where SCC decided case in
Stinchcombe that defense if entitled to al this
disclosure where they can properly ascertain to
case again to client and if they should go to trial or
resolve their charges and added benefit of plead
 DP victory lead to more CC approach to CJS
1. Stats = in majority of cases, most are dealt
with via plea bargaining g
2. Stinchcombe having facilitated in CJS
process
4. Third-Party and the Criminal Trial
a. Traditionally, seen as a boxing match between the Crown and the defense
i. Analogy of a 3 persons in boxing ring and 2 opponents and judge who is
represented as the ref
 Clear that the victim/ complainant is not involved in this process
 After the enactment of the Charter, victims started using the
Charter as a way to enter into the criminal trial process and
overtime, a number of cases decided that the recognition of other
third parties having a standing in this boxing ring and procedure
a. Challenge of thus view of being between these opponents
53

b. Where victims having entry into the criminal trial process


c. Media
d. Entry can be overstated
e. Situations where these third parties paining entry into the
criminal trial process
f. Change to how we see the criminal process playing out
g. Having the rights recognised and always balancing the
rights against that of the accused
ii. Example  Dagenais case and Bernardo
 Showing how 3rd party and entities in process cannot interrupt or
give full answers of defence and always balanced or tempered
 1st successful example of a third party into the trial was the
inclusion of media outlets
a. Dagenais and the CBC
 CBC claiming freedom of expression and victim
rights and how their privacy rights and how it was
in relation to freedom of expression under the
Charter
 Publication bans:
 “for justice to be done, it must be seen”: open court
principle
 Limits to when info cannot be published or if
proceedings are not made open to the public
 Case where publication ban and relation it has to
freedom of expression
 CC and judge using their discretion to ban
publication of details from media reporting on
details being set
 Bail hearings and trial
 In most if not all, the presiding justices will grant if
sought a publication ban
 Allegations discussed in courtroom or any evidence
given to bail hearing, the media cannot report on
what is said on the hearing
 Can report on release, but not on evidence
 Way to protect accused’s right to a trial
 Publication to protection fair trial later on
 Trial where evidence is called, that it will actually
be placed on evidence
 Often on the person identifying the evidence; in sex
assault cases, often ban on victim’s name and
complainant, but not on the nature of charges
54

 Publication bans are necessary in the criminal


process
 The judges in this case, authority to grant or
impose was challenged via media
 Media said that the publication ban that judge
sought to impose violates our right to freedom of
expression under 2b of charter
 Background
 Situation where religious members charged with
abuse in Ontario catholic school;
 Trial, cbc produced miniseries based on separate
sex abuse scandal and about similar case via
priests at this catholic school;
 schedule to be broadcast during actual trial during
that trial
 Defense brought application requesting a number
of diff remedies for what they saw the affront to
defense right to fair trial and asked for jury to be
sequestered prior to airing of sow and until
decision was reached
 Judge declined that and directed jury to avoiding
watching the show
 Day before the airing, the defense applied for
injunction for cbc airing the show and releasing
any info into until the trials were completed
 That injunction was granted, then appealed via
media to Ontario court of appeal
 Court of appeal upheld injection but limited it to
only applying to Ontario and Montreal
 Jurisdictional scope limited to Ottawa and
Montreal
 Appealed to scc
 Done via civil injunction
 SCC court overturned publication ban, and it was
in ban of freedom of expression under 2b of charter
 What case against Bernardo the court tried to,
recognized that other parties have role and rec that
sometimes third party, parties have a steak in the
process
 That trial judges to retain discretion to grant
standing and stuff to grant submissions and argue
procedure in trial process and to call evidence to
55

deal with eh issue of infringement of charter rights


pursuant to charter s2b
 What Dagenais did = gave third parties a
procedural steak in criminal trial and outlined new
framework for balancing competing rights claims
 What scc tried to do = create more pragmatic/
common sense approach for design with issues like
these at trial because of all the diff competing
rights
 Publication ban engaged privacy rights of
victims, fair trial etc.; what court tried to do= u
need to balance all these competing rights and no
clear winner and loser like you ill in a criminal
trial and need pragmatic approach
 Cant make every party happy all the time, need to
weight these competing rights if you’re the trial
judge and cant lose track of fair trial rights of
accused in these situations
 Other case @ third party entry in crim process
a. Bernardo
 Most sensation crim trial in the 90s
 Sensationalized bc video tap recording s were
uncovered of these women
 Issue= as part wanted to show videos of victims
and wanted to show it in the open court and wanted
the jury to see it
 No consideration of public to have view the
material
 Families of victims sought and were granted
standing in arguing if these video tapes should be
played in open court
1. Granted standing of party within the
criminal trial because of unique perspective
of if these tapes should be shown in open
court
2. Trial court accepted the increase of role
victims play and view as healthy inclusion
and goof to view victims, this way in the
COLLprocess
3. Consideration of victims should never
interfere with orderly business of the
criminal process or interfere with the
56

crowns ability to represent the public in


court
4. Getting to stand to see if these tapes should
be played wasn’t that heard looked at
Dagenais @ HOW TO deal with competing
interest sin the criminal process
 Dagenais was written to avoid clashes and fight
1. But it was still and adversarial process
 The media = one party and tried to say they have
right to see and report on evidence called
 Bernardo argued fair defense and trial
 Family argued that they would be harmed by the
public display of these viscous offences happening
against their loved ones
 Trial judge, drawing n pragmatic approach that
Dagenais talked about, the pubic in gall would be
able to hear the audio but not see video and video
would only be shown to jury, judge, crown and
defense and limited viewing of gruesome videos
 Respects open court principle (pragmatic
approach), respect of victims and the criminal
process)
 Pleased few despite it following spirit of Dagenais
 SCC rejected leave to appeal via fam
1. Civil proceeding field built didn’t impact
decision

5. Presumption of Innocence
a. Oakes and the Oakes test
i. The case asserted that an accused person is considered innocent until the
Crown discharges the onus of proving guilt beyond a reasonable doubt;
and even from the moment an accused is charged, until the trier of fact
establishes otherwise, you are to be presumed innocent
ii. Important in relation to Packer’s models
 CC relies on the presumption of guilt via experiences of police and
Crown in identifying factually guilty persons
a. Up to the Crown and judge to determine who they consider
to be factually guilty
 DP model relies on the presumption of innocence and one is
presumed innocent until the states proves your guilt beyond a
reasonable doubt and demands this beyond a reasonable doubt
standard for someone to be found guilty and gives the defense the
ability to find and have other factual guilt to be proven guilty
57

a. Presumptions of innocence = once of the golden threads


running through the criminal justice system
 One of the first cases dealing with this presumption
was R v Oakes
1. This case reflected the belief that
individuals are decent and law-abiding
members of community until proven guilty
beyond a reasonable don’t
2. Despite this presumption being an
important factor of the system, in reality this
presumption is lose among most members of
the public – once a person is charged, we
often take that with factual to legal guilt
beyond a reasonable don’t and society often
conflating these points
iii. Oakes
 Charged with drug offence (possession of drug for purpose of
trafficking), this offence was under the narcotics act, now CDSA
a. Under this act, number of different offences dealing with
possession of drugs
 Single possession  for your own private purposes
 Trafficking selling, giving, or others to persons
caught
 Possession for purpose of trafficking  person in
possession and State can prove that person had this
for trafficking purpose
b. Narcotics Control Act  once a person is charged with
possession and once State proved possession, that was
enough for proving possession of trafficking unless
accused rebutted such presumption and presumption that
once found guilty for possession and unless u can prove
yourself innocent and to pertaining of possession
c. SCC in Oakes, the narcotics and control drug substances
act violated presumption of innocence under s.7 and was
not reasonable under s.1 of the charter
 What the court found in relation to the narcotics and drugs
substances act = did not rationally advance war on drugs and
trafficking
a. Oakes demonstrates the advancement of DP rights and
reject CC logic and probability
b. The Oakes decision did not mean that any offence with
reformation offence automatically violated Charter and
presumption of innocence
58

c. Narcotic substance act did not lead to other aspects of


reverse of onus being struck
 i.e. possession of firearm  in the CC, it saying
that it is illegal unless you’re a licenced holder and
it is worded in a way which the Crown only has to
prove that you had possession of a firearm and
presumption of procession and possession without a
licence unless you can prove to court that you had a
licence. Those in possession of a gun must prove in
2possession of valid licence
d. also, rebuttable presumption in CC as pertains to driving
and driving offences
 under the CC, as it pertains to drinking and driving
offences, there are 2 within 1
1. operating a motor vehicle and have your BA
over the legal limit
a. having care and control over vehicle
when BA over .8-
b. for care and control, you just need to
prove that you had care and control
over the vehicle and in the driver
seat = display of car when in control
and must rebut this according to the
criminal code
c. this is also rebuttable position which
is a challenge and violates the
presumption of innocence, but us
saved under s.1 of the Charter and is
justified to combat the threat to
public safety when a person has
while intoxicated that may, rebut the
public at risk
 in broth cases, the courts decisions are based upon the concerns of
potential victims and decriminalization of victimization risk and
thought process and what’s an appropriate Charter right violation
a. Concern for victim rebut successful for presumption of
innocence
6. Substantive Criminal Law – Charter and murder provisions
a. Queen and Vaillancourt
i. Background
 The decision struck down what was felony murder where DP
was triggered via sympathetic person behind court
 Persons were at a pool hall and a friend shows up with a gun, a
fight ensues between friend and other customer; in this fight, the
59

patron is shot resulting in death while committing another offense


of robbery
a. A person who kills someone while committing the offence
of robbery is guilty whether death was intentional or not
 Person was caught at the scene and charged with culpable
homicide and charged under this section because they were
considered an accomplice under s.21 of the CC
a. Party provisions of the CC and being held liable for
criminal acts even if you’re not the person who throws the
punch and involved to a certain level, you are a party to
this offence
ii. Issue
 S.7 of Charter
a. Principles of fundamental justice under s.7 that an accused
person not to be held liable without showing elements of
subjective mens rea and must show that person knew that it
happened
b. Convicted of murder because robbers were responsible for
any other crimes occurring during the robbery and doesn’t
matter if he intended or didn’t intend for death even if it
was reasonably foreseeable
 Striking of s.2.3(d) looks at the act and punishment
a. Murder = automatic life sentence
 This produces stigma upon the offender
 Court found that moral blameworthiness must be
proportional to punishment a need beyond a
reasonable doubt of foresight
 Vaillancourt and no conviction because of objective
foresight
1. 214(b) no foreseeable death this section
was struck down by the SCC which required
mens rea on the basis of subjective foresight
of death (requirement of subjective
foresight)
7. Regulatory and Corporate Crime
a. Absolute vs Strict liability
i. Absolute vs Strict Liability (BC Motor References and mens rea and when
rights are on the line)

Actus Reus Mens Rea


Absolute liability  Only needs to  Because they only
prove act, not defence need to show you the act
(YES) was committed, the mens
60

rea is not required (NO)


Strict liability  Only need to  Once you’ve proven
prove act, but there is a mens, but allows defense for
defense (YES) accused to show defence of
due diligence and that they
took reasonable steps (NO)
Criminal  YES  YES
 HTA created absolute offense for driving with a suspended licence
a. All Crown had to do was establish conviction was that you
were doing the driving regardless if you knew that your
license was suspended
 No ability to argue that you didn’t know
b. Problem = successful conviction carried a mandatory
minimum of 7 days and if Crown proceed, actus reus, the
mandatory minimum was a prison sentence
c. This was challenged and made it to the SCC
 Argument  having an absolute liability offence
for an offence where mandatory min was jail
violated principles of fundamental justice under s.7
1. Holding that absolute liability offences were
violations of the principles of fundamental
justice were not saved under s.1
a. This decision and principles of
fundamental justice through the
requirement of fault component for
all offences were the possibility of
appropriate default component for
that offence
Lecture Four: Victimless Crimes
1. What is a victimless crime?
a. Packer  presence of a consensual interaction between two peoples
i. Result of Packer and DP measures was in the corresponding decline in
the use of the criminal sanction as it relates to victimless crimes – less and
less prosecutions based on victimless crimes which are based in
consensual transactions
b. Challenge of Packer’s theory  developed in a specific contextual time period in
the US
i. Rise of feminist thinking which arose after the development of these
models and before feminism became an intellectual force
ii. Significant changes to legislation and terms in which he uses to define
victimless crimes
61

iii. Part of the challenge comes from expansion to what harm has been
defined to be
c. Harm
i. Harm, referring to physical forms pursuant to physical violence against
someone
ii. Definition of harm is expanding and includes present forms of harm, but
also includes future risk against an individual, which includes
psychological and production of anxiety and fear to individuals
iii. Form of and expanded to include situations that contribute to social
inequality in society.
1. There have been correlations to explanations of harm and the
increased implementation of the sanction in trying to prevent the
occurrence of harm
2. Change in “Political Case”
a. Shift
i. Moved from classic case of the accused and their DP rights and going
against the state and inclination of it being CC oriented, move to cases
where DP rights are being pitted against victims and potential victims
ii. What we’ve seen over time is the result of a heightened likelihood of the
passing of new laws enacted by Parliament
b. Pluralistic politics of rights
i. This tried to describe that within each group purporting to respect
victim’s rights, cam controversial debates about the use of criminal
sanctions
1. Assisted dying and Sue Rodriguez
a. The politics behind support groups and what you saw was
3. Prostitution
 Packer criticized prostitution laws stating that the State was trying to deal
with morality by dictating what was right or not
 He saw prostitution laws as being scandalous as police officers need to
participate in prostitution in order to investigate (police posing as Johns,
sex trade workers etc.)
a. DP prior to Charter
 Most successful DP challenge before Bedford came before the
Charter was enacted; the Hutt case
 Hutt  JJ held that the only pressing and persistent behavior of
sex-trade workers created a nuisance; ruled that it was not a
nuisance
o DP helped decrease the use of the sanction and the SCC
expressed concerns about the assembly line process of
trivial punishments
b. Changes post Charter
i. CC affirmed through new legislation  3 offences codified into the CCC
62

1. Prohibition of publicly communicating for the purpose engaging in


prostitution
2. Prohibition against keeping a common bawdy house – prevented
sex trade workers from offering sexual services at locations such
as brothels or their residences
3. Prohibits living off the avails of prostituting (including but not
limited to pimps)
ii. Changes to law instead of social policy
1. Codified due to CC perspective— our streets are being overrun,
we need to take our society back
2. Criticisms
a. Very punitive, burdened those who were most vulnerable
(the sex-trade worker is the most disadvantaged) and the
sanction fell of them; based on the criminalization of
politics; ignores the root of the cause of prostitution and
promotes the cycle of violence
b. Feminist groups opposed this  wanted to deal with the
root of the cause of prostitution
3. Effects  increased CC activities such as arrests and
prosecution… in crease in court resources being used; CC being
used to justify legislation
a. SCC 1991 entrapment
i. SCC opposed prostitution laws to help with the
nuisance concerns
ii. Random virtue testing – arresting in Vanier
because of location
iii. Bedford
1. Legal challenge
a. It was a legal challenge, challenging Canada’s prostitution
laws that had already been challenged in the 1990’s but
the court upheld the law
b. The three laws discussed above were challenged by three
women on the basis that these laws prohibited prostitutes
from implementing security measures - SCC said the three
provisions did not pass Charter scrutiny and infringed
Section 7 rights of prostitutes in a manner that is not in
accordance with Section 1
c. Looks at their decision in 1990 and says they’re not bound
by that decision they previously engaged in - These
conditions, these criminal laws and offences impose
dangerous conditions on sex trade workers – they
prevented people in a risky but legal activity from taking
steps to protect themselves from risks - Negative impact on
63

their safety and their lives – Parliament’s intention to deal


with the issue of public nuisance is unjustifiable
4. Drugs
a. Marijuana
i. Mandatory minimums were not helping because it was not doing anything
but keeping young people in jail and holding them back/running their
lives
5. Gambling
a. Gambling has become much more liberalized because the government makes
money from it - Criminal sanctions are used to ensure that they are the only ones
(monopolizing) controlling that money - It may not always be a consensual
transaction for addicts … they need certain protections put into place
6. Abortion
 Criminal sanctions on abortions made abortions expensive and dangerous -
Packer championed for total decriminalization without dealing with equality
a. Morgentaler
i. SCC used Due Process to restrict this sanction
ii. The repeal of abortion laws in Canada was the only good result to come
from SCC
iii. Attempts to reintroduce the Re-criminalization – even to this day
iv. Expansion of harm used by courts to put people away
v. After the Charter came into effect, announced plans to open abortion
clinics in  Winnipeg and Toronto
vi. Charged with conspiracy to commit crime without committee approval
vii. If you wanted an abortion, you had to apply to a committee in your
jurisdiction and the committee would decide whether to approve this
application
viii. A jury acquitted Morgentaler, however, law was upheld at trial that he
acted in violation
b. use of the criminal sanction
i. SCC struck down criminal sanction concerning committee approval and
found that delay and uncertainty with this process violated Section 7
against women
ii. Criminal sanction was used to protect those who were seeking services of
Dr. Morgentaler
c. continued attempts to re-visit issue
i. After decision, there were attempts to reintroduce criminal sanctions that
were defeated
ii. Criminal sanction was used to protect those who seeking services of Dr.
Morgentaler
iii. Government tried to re-enact abortion as an offence (portrayed foetus as
victim) (does life start at conception or does life start at birth?)
iv. Court allowed for injunction against protesting outside abortion clinics
because it would cause State imposed harm
64

v. Psychological harm against person going through an abortion


vi. Courts sided with psychological harm over Freedom of Expression
7. Assisted suicide  Should not matter whether you are disabled – does not affect the
individual’s choice to die with dignity; Criminal sanction based on that allowing people
to kill themselves will mean everyone will want to do it
a. Rodriguez
i. Victim of ALS (Lou Gehrig’s Disease) and asked for exemption from
Criminal Code dealing with assisted suicide
ii. She wanted assistance with suicide once her disease would have worsened
to the point of immobilisation
iii. SCC declined to give her and her health care providers that exemption …
majority conceded that offence will cause Rodriguez pain and stress,
thereby violating Section
iv. 7 saved by Section 1 Criminal sanction upheld due to moral belief and
the policy was supported by community values
v. Court justified decision by appealing to victim’s rights stating that there is
the possibility that people of disability will be taken advantaged of (using
victim’s rights to maintain crime control)
vi. Majority belief that criminal sanction protected vulnerable – they invoked
a “slippery slope” argument
vii. 75% of Canadians agreed with Sue Rodriquez’s position
b. Gloria Taylor
i. Carter v. Attorney General challenged the prohibition of assisted suicide
including Katie Carter and Gloria Taylor who suffered from ALS
ii. Court struck down on Section 241 and found that Section 241 violated
Section 7 and was not saved under Section 1
iii. Court found that trial judge who struck down decision was not bound by
Rodriguez decision
iv. Section 7 rights differed from those in Rodriquez especially the issue of
whether the law was disproportional
v. Trial judge looked at several countries who legalized assisted suicide and
they rightly balanced concerns of disabled
vi. In ruling, court said, person who is contemplating decision because
assisted suicide is illegal, should decide to end their life prematurely and
often violently or suffer immense pain and die slowly
8. Hate Propaganda
a. Summary
i. Only if there is very specific proof, act, intention and motivation (the
wilful hate is done deliberately)
ii. Not enough to just say something hateful
iii. Came from our ratification of UN document discussing hate speech
iv. Few prosecutions happen… Attorney General must approve and must
prove wilful intention to promote hatred
b. R. v. Keegstra: 1984 willfully promoting hate against Jewish people
65

1. Section 318, 319, and 320 deal with the issue of hate speech
2. SCC upheld criminal code provision of hate speech – saved by
Section 1
3. Keegstra attempted to use Section 2(a) and was only fined 5K o
SCC recognized psychological harm caused by hate speech o
Justice McLachlin rejected clear and immediate threat argument o
Risk discourse was merged with rights discourse – involves
violence and discrimination o Acceptable to use crime control to
prevent risk of harm
c. R. v. Zundel:
i. Attorney General first denied prosecuting
ii. Zundel believed the Holocaust was a hoax
iii. Private prosecution brought by citizen and then the Attorney General took
over
iv. Zundel was convicted and given a 15-month imprisonment
v. Exposure to trial caused secondary victimization due to cross examination
vi. Zundel decision was overturned and became a subjective fault
vii. Zundel honestly believed the Holocaust did not happen
viii. SCC decided that the law was unjustifiable and violated Section 2(a)
ix. Stuck down law stating that you could not spread false news – SCC said
even lies were protected forms of freedom
x. Freedom of expression is the main component of this issue
9. Pornography
a. Packer believed obscenity laws were akin to prostitution – simply an enforcement
of morals… Court should not concern itself with this
i. Parliament attempted in 1980’s to introduce obscenity
ii. Many groups said criminal law was not enough to deal with harms
created by pornography
b. R. v. Butler:
i. Owned a video store and had pornographic material in his store
ii. Charged under Section 163 of the Criminal Code
iii. Trial judge acquitted because parliament could only outlaw violent or
dehumanizing material at SCC, some feminists said pornography was a
form of hate propaganda against women
iv. SCC upheld obscenity laws, justifiable limit of freedom of expression
v. Court created three categories of porn:
1. Explicit sex with violence
2. Explicit sex without violence but with degradation
3. Explicit sex without violence or degradation
vi. Violence also includes threats
vii. Courts stated first category will be against law, second category could be
undue if risk of harm is substantial, third category will be tolerated unless
it employs children
viii. Aftermath of Butler – parliament legislated against child pornography
66

Lecture Five: Women


1. Introduction
a. Feminist groups have played an important role of potential and current criminal
victims in respect to victimless crimes and also in respect to victimless crimes and
crimes of violence
i. These groups are not united in approach to victimless crimes (i.e.
prostitution)  decriminalization vs the sanction
ii. More consensus amongst these groups in relation to new laws and
strategies to respond to sexual and domestic violence towards women in
society
iii. Perspective of violence against women has changed over time
1. In the 70s, you see the start of domestic violence being regarded
as an issue in society and concern leading to raised awareness,
education, and harm reduction (crisis centers, domestic violence
shelters)
2. In the 80s, shift to how society views issues of domestic violence
and violence against women
a. Legal reforms and call to action for the government to
makes crimes against women easier to detect, report, and
prosecute
iv. Legislative changes
1. Subject to DP changes in the 90s and 2000s, what we see is less
success in defending these reforms than that of victimless crimes in
the 80s and 90s
a. Pattern emerging = feminist-based court parties losing
battles in courts but winning in regard to laws being
created by parliament in response to these perceived issues
b. Success in these reformations were not met by parliament
by equal payment, childcare, and poverty reduction
i. What this leads to = the concept of criminalization
of politics alive and well and parliament dealing
with issue via reforming criminal laws instead of
looking at underlying issues and causes which
contribute to new laws being created
ii. Instead of looking at the issue via a holistic
approach, it is used as a pervasive measure to
violence in society
v. The use of the criminal sanction
1. Criminal law being used to respond to concerns of feminist groups
and criminalisation of gender politics including sexual assault
laws which set the stage for new political cases which made it up
to the SCC
2. Female Criminality: Due Process?
67

a. Battered Women and Self-Defense


o One response to dealing with the issue of female criminality was to
argue about the accused’s past victimization which could be relevant
to see if she acted in self-defence
 This concept of battered women and self-defence was an
attempt to make DP work for women, but didn’t work in the
final product (i.e. in the case of Lavallee)
 More liberal in way of dealing with these provisions but didn’t
really change much to see how women cycled through the cjs
 CC model and having women pleading to crimes that weren’t
handled properly
i. Walker’s Cycle of Violence and Learned Helplessness
1. Walker’s Cycle of Violence
a. By theory, increased knowledge that women as victims
i. Both influential and controversial at the same time
ii. Walker and 3 stages to explain why women will
stay in an abusive relationship women stayed in
relationships because of system of helplessness; a
person in a domestic relationship where they were
subject to violence will come to the conclusion or
belief that there nothing out there to help them and
they’re stuck and need to deal with it themselves
iii. This reverse could support mandatory arrest and
prosecution for domestic violence victims and used
even if against wishes of the victim
iv. Walker’s 3 stages of violence and learned
helplessness
1. Tension building
a. where she tries not to trigger anger
2. Acute and unpredictable
a. Where females are subjected to
unpredictable battery, the recipient
would not know when next episode
of violence would be inflicted upon
her
b. This is what leads to helplessness
3. Kindness and loving behavior
a. The male states that he will never do
it again, this ensures power over the
relationship
v. this theory was sued to support mandatory arrest
policies and mandatory minimum prosecution
68

against men who abuse their partners, even against


the woman’s (victim’s) wishes= Crown’s policy
ii. Self-Defense Provisions: then and now
1. Charges are changed dramatically from conservatives in 2010 and
2012
2. Section 34
a. Case of Lavallee where death is caused and this is second
part of it, it is justified anything justified under being
objectively reasonable
b. Objective vs subjective reasonability
iii. Lavallee
1. Background
a. Charged with the death of her husband, she shot him in the
back as he walked away from her
i. He tells her that he was going to kill her once all of
their guests left that evening
2. Section 24 and 220 4(2)
a. The assault wasn’t currently happening when she killed
him and wasn’t happening during threat of further
episodes of violence
b. Self preserving = could have been her leaving house, made
call to police, and could have used other methods which
didn’t result in her killing her husband
3. Proceedings
a. Trial judge
i. In actual defense, the defense called an expert
witness on battered women’s syndrome saying that
Lavallee was subject to numerous cycles of violence
and was subject to extreme tyranny from the
evidence
ii. Trial judge allowed the testimony and told jury to
give it only minimal weight in their application 
their deliberation resulted in an acquittal
iii. This decision was appealed to the Ontario Court of
Appeal
b. Court of Appeal
i. Appeal found that the findings of battered women’s
syndrome to be relevant to sentencing is not
relevant to how self-defence ought to be applied
c. SCC
i. No interest groups intervened in this case and SCC
reversed mattered and found the expert testimony
as to how s.34 would be applied
69

1. Applying 34 to fact scenario, one would


conclude that it ought not to have been
applied
2. Expert put the jury in the shoes on Lavallee
and other battered spouses
ii. Looking at the mattered through the perspective of
a battered woman, s.24(a)(b) would be applied
1. Someone being subject to cycles of violence
and jury findings that husband telling her
that he would kill her lead her to believe
that this would happen, and she was
reasonable under the apprehension of death
2. Learned helplessness and explanation under
sub (b)
a. Years of this type of behaviour
where she would have left the
relationship and explained why it
was not only subjectively
understanding but also objectively
reasonable
iii. SCC relying on premise of battered women’s
syndrome and apprehension of death and imminent
harm
1. Consider history and perception of events
via accused
iv. Majority decision
1. Justice Wilson
a. Attempted to quantify image of
Lavallee as a helpless woman, who
had no job, kids to care for, and
living in fear of retaliation
i. Studies where parties are
separated and elevated rates
of risk of violence against
women
v. On the flip side, the SCC was quick to warn people
that just because someone was suffering from
battered women’s syndrome, that they were not
always subject to acquittals and not an excuse to
kill someone  there was worry from public that
this court case would result in women killing their
partners and getting off on it
70

1. The news highlighted this that men were


worried that this decision would provide
justification for someone to kill a person
just because of abuse
d. Aftermath
i. Not many cases benefited from the defense of self-
defence because of being in an abusive
relationship, as outlined by the Lavallee decision
and how being a battered woman was only a
mitigating factor
ii. What we see over time = battered women’s
syndrome being used as a mitigating factor in
sentencing
iii. Study post LaValle  only 3 out of 98 would have a
Lavallee sort of decision of self-defence-based
arguments made
1. Enabled and legitimizes guilt with holding
out the promise of an acquittal
2. Suffering abuse in relation was used in plea
bargaining from murder, to manslaughter
3. Instead of providing justification, used
within the CC assembly line to keep the
conveyor belt moving and SCC decision
didn’t result in a whole bunch of acquittals
a. The opposite occurred  less
women used this dense as a way to
reduce their charges and time spent
in custody
3. Wife (Spousal) Assault: Crime Control or Victim’s Rights
a. Packer
i. Packer and the assumption that greater efficiency in processing reported
crimes would control crime; the better you became at prosecution CC
offences reported, decrease in actual crime happening
b. Victimization survey
i. Victimization surveys have become an important tool to achieve important
reform to substantive criminal law, punishment, and evidence
ii. In Canada, we have official stats of crime rates, how the charges were
prosecuted and how many were acquitted etc.
1. What the surveys reported were not limited to instances of
reported crime
2. States only representing reported crime, victimization survey
shows how much unreported crime there was out there and how
thee states were sensationalized which resulted in calls for action.
71

a. Could have changed cjs and sanctions


b. High unreported crime constricted for why we need
changes because its premised on the belief that no one
should not report crime and if people don’t report crime
then the system is broken
iii. One of the most important victimization surveys impacted law and
legislation to women
1. The survey reported that 1 in 10 women who were in a
relationship were battered and were in one where they suffered
physical violence and that number became a magical figure and
didn’t require any real policy response and could be used to
understand reasoning
2. 1 in 10 interpreted to justify a new response
iv. Response from Parliament
1. Parliament amended the criminal code to facilitate new arrests via
s.495
a. 495 is applied through a lens of police powers under
arbitrary arrest and powers; this section was enacted by
Parliament to assist police in resolutions
2. Prior to s.495
a. Police before this could only make arrests when they saw
the incident happen and obvious domestic violence which
occurred behind closed doors. It allowed police to have
reasonable probable grounds to arrest someone when
belief that they will complete an indictable offence
3. S.495
a. Mandatory arrest policies and reasonable probable
grounds for arrest
i. Mandatory arrests even if against wishes of victim
and complaint
b. Crown Manual amended for zero tolerance policy against
domestic violence
i. Walkers Cycle of Violence helped form the Crown’s
policy manual  these victimization policies
enacted a zero-tolerance policy that laid an
assumption that by enacting these types of
prosecution would eliminate partner violence
ii. This approach dealing with perceived societal
issued through criminal law facilitates the
criminalization of politics and domestic violence
not seen as significant societal issue and how
parliament responds = different reforms
72

1. Example  ministers’ response for status of


women was that of refusing to implement
recommendations for reforms made at that
time; instead implemented a CC approach =
zero-tolerance approach to crime against
women
a. How the use criminal law to deal
with issue of taking issue for CC
oriented approach to problem was
perfected by removing victim from
process
i. Policy and procedure, once
you remove the victim from
the process, they suddenly
become marginalized by
initiative made in their name
ii. Victims rights now being
used as a method CC
strategies in our justice
system
4. Sex Assault Law Reforms: Victims Rights\
a. Victimization surveys
i. How they played an important under reporting of sexual crimes in
Canada and other influential studies documented to rates of police
charging of found in cases of sex violence against women
ii. What was found was essentially that when complaint was made by the
individual, police often finding no reasonable and probable grounds to
continue case and often would close it and determine it to be unfounded
iii. Rates of conviction of sexual violence offences and how these conviction
rates were lower than in other criminal cases outside of sexual violence
iv. Prior to 83’, rape was the offence and police finding how rates of rape
and conviction were lower than other forms of criminal activity
v. Today aftermath of Ghomeshi case, police rates of founding reasonable
grounds for sex assault lower than any other offence and smaller with
conviction rate currently
vi. Same studied relied upon in 80s in cjs are still being used to make
substantive changes to CC and how offences are prosecuted and defenced
b. Changes in law from 80s to now
i. During this time period occurred reforms where offence and rate of
indecent of assault was replaced with 3 categories of sex-based offense
under (s.271,271, and 273)
1. Sexual assault
2. Sexual assault with weapon
73

3. Aggravated sexual assault


ii. These changes made did not make the sanction any more punitive, and in
fact, these changes allowed for offence of sex assault to be prosecuted via
summary conviction where summary conviction was reduced to max of
6mo.
1. Some feminist groups argued that lower penalties spoke to society
that crime is no longer as serious as it once was
2. The government in changing the offences of rape to sexual assault
and decreasing max punishments in some cases
a. They thought that a less punitive approach = better
upholding of integrity of victims
3. Cases prosecuted increased, but punished more leniently than
those of the past
a. Law reforms made in 83’s in regard to sexual violence
against women and changes to evidence law in Canada
i. Example  laws of evidence changed to how it
pertains to situations where a victim or
complainant would give evidence with no
corroborating evidence
1. In the past, if a woman were to come to
court and in allege that a and b occurred,
and in relation to issue at hand, the judge
would conduct proceedings in a manner to
which they would instruct the jury to not
take allegations at face value if evidence
was not corroborated
4. Current
a. Now under s.274, it enacted to say that if an accused was
charged with sexual based offences, the judge…
i. S.275
1. Recent complaint appealed
b. Essentially, if a person was delayed in making a complaint
against them, that the delay in the past could be used to
attack their credibility
i. stereotypes that if this violence happened then, then
there shouldn’t have been a reason in their delay to
making a complaint
ii. Law reform of 83’ repealed component of recent
complaint doctrine and no effect on reliability of
individual
c. S.276 and rape shield protection  past sexual history is
inadmissible
74

i. Relevance of sexual history was connected with the


Papa-John defense (honest, but honest mistaken
belief defense)
1. This honest, but mistaken defence and after
this, judges had to instruct jury to how
reasonable this honest but mistaken belief in
consent was
2. Later on, in Parliament, requirement
accused to prove they took reasonable steps
in achieving consent
5. Today
a. Todays tide-and-change to CC relating to sexual history
and third-party records that come on the heels of reports
and studies and believe hat you can still cause a decrease
in criminal offences committed against someone if you just
change the law appropriately
i. Something that comes about these changes = these
cases are put in front of the SCC if the accused’s
rights are being violated (amendments to the CCC)
5. Due Process v Victims Rights – the New Political Cases
a. Seaboyer – “rape shield”
i. CCC
1. S.276  law that limits the ability of the defendant to cross
examine a complainant of sexual assault about their prior history
a. Absolute bar of asking people their histories and
susceptible to Charter challenges because it’s a significant
thing for the courts and Parliament to take away from full
answer for the defense
ii. Decision  the court found that the evidentiary restrictions were
unjustifiable in giving full defense and was contrary to s.7
1. Justice McLaughlin
a. Principle that innocent persons must not be convicted
i. How its one thing to restrict freedom of the press,
its another to restrict an accused’s persons defense
 how a factually innocent person should not be
sacrificed from reporting or for a successful
prosecution
b. The decision trying to devise a middle ground to enact
guidelines to protect against sexist reasoning as contact
i. Twin myths that Parliament sought to prevent
2. S.276 and how its built within the section that past sex histories =
inadmissible and is being less worthy of belief
75

iii. Seaboyer and concept of risk played a role in both majority and minority
judgements
1. McLaughlin and majority = innocent person being found guilty
focus on the conviction of a factually innocent person
2. Dissent and Dube = concern on risk of someone being a victim to
sex assault
iv. Media
1. Media and certain individuals saying that its re-assaulted women
and re-victimized them
a. This resulted in anecdotal reporting
b. Media reaction causing Parliament to respond, one of the
things suggested by Parliamentary sub committee that the
rape shield should be re-enacting using s.1 of the not
withstanding clause
i. Ends up enacting Bill C-49 outlining rights of
accused and victim
1. Recognition that prior sexual history may be
relevant in some circumstances and
modifying the Papa -John defence for the
accused to take reasonable steps to see if
consent present basis for what 276 is
today
2. Essentially a whole system in place to guide
judges for when prior sexual history could
be used and significant gate keeper function
in place to protect complaints form potential
attacks on their prior sexual histories
3. Rare that a 276 being applied because of
procedural hurdles and how in relation and
more times than not, not being relevant and
courts recognising this overtime
b. Daviault – “intoxication defense”
i. Background
1. Case where we see DP rights for accused vs victim or potential
victim
a. Situation where Daviault becomes extremely intoxicated
and is hanging out with his wife and friend of his wife (she
is a paralyzed 65 y/o)
b. Daviault got drunk almost to the point of automatism and
violently assaults the woman
ii. Defense  testifies that he drank so much that he didn’t know what was
happening and was in an automatism-like state and this was the basis of
defending the crime which requires general, but not specific intent
76

iii. Appeal  ruled that intoxication until state of automatism is not


applicable
iv. SCC issue before the SCC is that of if an individual is in an extreme
condition of drunkenness; similar to that of automatism, is a bided of
defending crime which requires general, and not specific intent
1. Ruling
a. That definition under s.7 could not be saved under s.1, and
afforded him a new trial and ruled that intention for
intoxication is replaced for the commission of a criminal
offence
b. Limited to intoxication
c. Established that defense inly was on a balance of
probabilities as the accused would have to prove reverse
onus
v. Media outrage
1. Reaction
a. Reaction made for quick legislative reply and instigation of
reaction to case and introduction of Bill which made it an
offence to commit a sexual offence while intoxication
b. Bill C-72 took away the defense of intoxication for violent
offense and that getting drunk and doing things beyond
your control marked a departure from standard for
reasonable care
vi. Defense counsel vs feminist groups
1. Punishing someone for not having a guilty mind and substituting
form of criminal intent
a. New risk at forefront here
b. Enacting new law getting party approval and victims rights
for DP response for a DP victory
i. Section enacted; you will see that the defense is
significantly limited
ii. If you go out and knowingly get drunk and
intoxicated, even if you didn’t have the intention,
the intention will be substituted here
rd
c. O’Connor— “3 party records”
i. Background
1. Situation where you have a Bishop accused of raping Indigenous
students and defense sought to use school and health records as a
way to counter historical allegations
2. Without having heard from the victims, the judge ordered the
victims to give complete copies of info to the judge and defense
(disclosure and Stinchcombe) and Crown refusing to disclose
certain records
77

a. Stay of proceeding occurred and was revered via court of


appal
ii. Decision  that medical and counselling records for complaints held via
third party can be disclosed in 2 ways:
1. Show likely relevance  accused through counsel has to show
judge why records are likely relevant to issue at trial
2. If judge finds likely relevance, judge will take and review evidence
to determine what records will be disclosed on a balance
considering the privacy of the victims and the rights of the accused
to see evidence against them to be able to give full answer and
defense
d. Carosella
i. Background
1. Facts
a. Complainant consulted sex assault centre for how to lay
charges
b. Counselor made notes throughout meeting and eventually
after meeting concluded, destroyed the notes
c. The accused didn’t know that the notes over the meeting
would later be shredded, he was planning on giving them
to the judge for vetting to be submitted as evidence
ii. Decision  the judge ruled that the destruction of evidence and because
it was destroyed
1. SCC upheld this decision and said that this destruction while
police interfered to individuals right to answer and defense and of
a fair trial
iii. Media
1. Media attention to O’Connor and Carosella was not front-page-
news and limited ripples to society
a. After decision in O’Connor, the government enacted
legislation that responded to the ruling
b. Globe and Mail criticism at government for changing laws
every time a specific interest groups raise concern
i. S.276 and direct reflection of O’Connor
c. Revered more serious requirements of O’Connor and
judges view of further rights and lessoned some
2. Stinchcombe s.278.1
a. Stinchcombe and definition of record as changing
i. If accused person wants possession of record, they
need to go through application process
ii. Look at sub section 2 under 278, this section
changes the requirement of crown under
Stinchcombe
78

1. Record remains in Crown’s hands unless


complainant or witness consent to record
being disclosed to defense
2. Mist give defense notice of records, but
don’t need to give it to them
a. If you want to get records, you need
to do a pre-trial motion in which you
give formal notice to Crown,
complaint and or holder of the
record in addition to setting out why
these records are relevant on the
grounds which are insufficient to
own but need reasoning— need to
show why you need the disclosure
6. New Amendments
a. Bill C-51, 273.1 and 278
i. How public discourse about rights of complainant vs rights of accused are
rising post Ghomeshi trial and standing with victim and suggestions that
presumptions of innocence should be changed in sex offences along with
rules of reliability and matters should be changed and that new process
and laws that should be applied
ii. In Bill C-51, some say it does nothing and government has started a
nightmare and number of different sexual assault reforms made
1. Example  issue of consent and mistake of belief of consent is
amended in the Criminal Code
a. 273.1  a person can no longer consent if unconscious
(Queen and JA on mistake of law or if belief is based on
complainant’s passivity and if complainant did not say no
even if history of i.e. conduct without saying no and now
all of a sudden in the last one there wasn’t?)
i. Ewanschuck
2. Issue  the issue where you will see Charter applications being
brought was in respect to the rape shield law under s76 and its
now being amended to include that past sex activities and contact
(emails, text, etc.) are all example of prior sex activity and now
subject to a 273 application
iii. Third party records under 278 also changed, now if you need to make an
application and born out of suggestion that complaints in Ghomeshi trial
and texts being used against them
1. One way to test allegations are the reliability and credibility of
complaint
79

Themes:
1. Criminalization of politics
2. DP vs CC
3. Emergence of DP and how CC measures are causing DP measures to retreat
4. How DP is for CC
5. Legislation and law making: the courts vs parliament
6. How Packer’s models are incomplete and what perspectives are missing
7. Why Packer and his models have withstood the testament of time so well
8. The history of DP in Canada and how the US and their SC decisions influenced our law
making (Bill of Rights and Charter)
9. Imp landmark cases pre and post Charter and what they mean for DP and CC in Canada
10. How Canada went from a dominant CC model to DP, to now a retreat
11. Changing crime states, and creation of areas of disproportionate crime because of
parliament changing legislation to eliminate a certain criminal offence from occurring
12. Give examples of victimless crimes. Use Packer’s theory to describe why these are
considered victimless crimes
13. What were the two crime control models and what are the two new models being used in
society, today?
14. What rights so sections 8, 9, and 10(a)(b) protect? What are the three deciding factors for
excluding evidence
15. What is evaluated when considering if the delay in the court process is reasonable? There
are two important cases during this time, choose one and explain what they were arguing
16. Why does Packer argue that in ways the Warren era was indicative of how due process is
for crime control and how the administration was largely unsuccessful
17. Contrast quasi vs absolute exclusionary rules
18. How was police misconduct handled pre and post Charter and DP revolution changes
Roach and his argument that the traditional models of crime control are inadequate and can no
longer explain the law and politics of criminal justice
 He argues that victims’ rights have a significant impact on the dimensions of
criminal justice policy
 Argues further that criminal justice reforms that have come to dominate the
political agenda because of their symbolic wright because they are “relatively
inexpensive” compared with other more structural reforms
 DP and CC and critique, the proposal of new grounds for new models of
criminal justice which would include the rise of victim’s rights discourses,
accounting for unreported crimes and the development of restorative justice
practices
 New model of victim’s rights which fuse DP and CC and integrating rights
and voices of the victims
 Policy shifts in the criminal justice system which lurches from DP
developments such as the development of the Charter rights of accused
80

persons and pre-accused such as drunk drivers, to CC accommodation such as


tele-warrants
o How public perception of crime is shaped and often mobilized through
media reporting on selected leading cases, the publicity attacked to
each, and media statements by legislators
 Shadow crime = often overlooked and underreported crimes dealt with via
informal and extra-legal measures
o Such measures include confrontation, shaming, community
involvement, insurance claims, and informational reconciliation
o This model contextualizes accountability and responsibility for crime
Additional notes
- Faint hope clause?
- Make sure to review case law
Case law (not complete)
 lecture one  cases which came from Charter and the cases from states under warren
that were important
 Lecture two  R v Grant (exclusion of evidence), r v. Suberu (detention), r v St Cloud, r
v antic, r v Fearon
 Lecture three  r v Lahiry, r v Godin, r v Jordan, r v Cody, r v Picard
 Lecture four  r v Bedford, Oakes, Morgentaler, Rodriguez and Carter
 Lecture five 

Lecture six  youth

- Background
o Calls for more punitive reaction to reduce risk of victimization (CC) and is similar to the
new political case as seen in Seaboyer, Carosella, and O’Connor is true for young persons
and how cases seen are pitting the DP rights of accused to the rights of victims and
potential victims
o YOA (+) level of imprisonment beyond what was seen in the YDA
 YOA = unmitigated disaster b/c of reliance on punitive measures which (+)
punishment
o Young persons from perspective of victims or potential victims
 One antiquated element Packer argued was of statutory rape that they were
imaginary crimes which should be repealed
 Issue of trauma prevalent which is different from that of adults
o 1970s’  not much terms of reference to abuse whether it was sexual or physical against
young persons and precipitated changes to procedural and substantive law
o 1980s’  reporting of sex abuse skyrocketing
 Historical sex abuse cases: Dagenais and CBC, St- Vincent, O’Connor, cases that
were happening more in the 80s; and issue of reporting increase in demand from
criminal law and sanctions to deal with such societal issues
 Victimization used to ask for criminal sanction to reduce risk to deter ppl from
committing criminal offences against young persons
81

- Sex abuse: victims rights and CC


o The Badgley Report (1984)  dealt with unreported sex violence against young persons
 Findings: 1 in 2 females subject to unwanted sexual contact, 1/ 3 females subject
to unwanted sexual acts (from minor to moderate etc.)
 Reveal that 80% victimization occurring when young persons
 75% and 90% of males kept it secret from others these stats used to
argue that the victimization survey and report afforded more right to
victims and protections to them
 Premised on idea that children and young persons have absolute right to
be protected against such offences and that society has duty to protect
them
 Shift from micro to macro view and focus of community and issues
 High rates of victimization and unreported crime = failure of the criminal
sanction
 Recommendations made b/c/ of report which included violence against children,
prostitution and child pornography
 Changes created were of the creation of gender-neutral offences, belief or
position of statutory rape and dealing with it outside of criminal law w/
counselling, change to Canada Evidence Act re: to do away with
traditional understandings of distrust w/ children
o That competent children should be able to testify, abolition of
corroboration requirement, committee also sought to loosen
rules around hearsay, and publication bans
 Little concern by victims to the need of having to deal
with our substantive criminal law
o Much of the considerations = Charter proof, advocating for
heavy punitive measures w/ little consideration to what victims
needed and wanted  same persons subjected to this
victimization were also petitioning more holistic approaches like
counselling outside of the trial and calls for punishment and new
offences created w/ minimal emphasis on hos society treats
young persons
o Legislative responses  painted pic of our CJS in disrepute and one that didn’t deal well
w/ victimization
 Bill C-15 in 86’
 Focused extensively on changes that the report called for and premise
was the acknowledgment of failure of CJS
 Substantial changes to CC which included: new sex offences re: young
persons, changes to law on consent, creation of new offences like sexual
interference, prohibition of indirect or direct touching, sex exploitation,
mistake of age no longer a defense unless you went through the
reasonable steps to verify it, made testifying easier for child witnesses
(i.e. use of a screen), allowing them to be on the stand in different room
 Critique  didn’t focus on all forms of abuse as pertaining to young
persons
o Neglecting to deal with societal and socio-economic issues such
as poverty, homelessness, and hunger, classic example of how
the criminalization of politics and criminal issues receiving more
attention and that poverty cannot be solved through criminal
sanctions
82

o Evidence counter-revolution response to report and motivated via judicial attempts to


facilitate the prosecution of child sex assault cases
 Khan  seminal case dealing with the SCC allowing the Crown to call hearsay
evidence b/c seen to be both reliable and necessary
 Evidence no longer available and only means to communicate it was via
the admission of hearsay, abandoning traditional distrust of children and
says they the testimony of young persons like any other complainant or
victim
 Creation of s.715.1  Crown allowed to take video statements and play it as long
as the defense can cross examine it
 Crown cannot play video of someone and must give evidence in Chief
 Section allows the video statement to be played as long as the defense can
cross-examine it
 Challenged in Queen v L(DO), but was upheld because the child witness
had to still adopt the statement under oath to allow defense to cross
examine
o Screen also used as testimonial aid
o SCC stressing the need for multi-faceted considerations for
witnesses when ascertaining the truth of what happened and
considerations for the accused
o Pre-charge delay counter revolution  dealing with delays in reporting victimization
 Recognition by courts that just because significant period of time has elapsed
doesn’t = less worthy of trust
 Courts allowed to proceed on historical accusations of sex abuse
 L(WK) where trial court granted stay of proceedings b/c delay in
reporting
 Decision = victims don’t always report asap and is a stereotype of
victimization, forcing them to do so before they’re ready isn’t right
o If not allowing the prosecution of cases b/c not reporting = de
facto limitation period and how perpetrators could rely upon this
for lack of reporting
o Statutory rape
 Prior to 88’, was an indictable offence to have intercourse with youth underage of
14, regardless of consent or belief of age
 Committee report recommend that this absolute liability offence be retained on a
CC basis
 Detection and prosecution = deterrent
 Extend to girls under 16 y/o and necessary to protect them from
emotional ham, exploitation, and pregnancies
 Law = effective deterrent
 New law  prohibited any sexual touching of children under 14 and
allowed accused a defense that they thought child was older only if they
took reasonable steps to ascertain age
- YOA: DP and CC
o YDA  that young persons are to be treated differently than adults
 Concept of “loco parentas” where the state acts like a parent towards youth
involved in criminality, believed troubled youths were misguided and needed to
be re-directed
 Imp = state intervention didn’t gave to wait for a criminal offence had occurred
 S.38  DP not a concern b/c the premise was that the state would always
treat the child fairly and act in their best interest
83

 Disposition to treat them not overturned and was done via perspective of
parent over child
o Act operated as a coercive CC assembly line, like a criminal court
w/o protections
 DP finally coming into light and if state actually can look out for best
interest of child
o Adversarial process didn’t mesh with principle of loco parentas
o Recognition that youths needed unique protections, led to
development of YOA
o YOA
 Developed via fed government and introduced under new banner of rights and
responsibilities
 More flexible sentencing to promote reconciliation between offender and victim
 More victim non-punitive approach and that they should receive more
protections
 Waiving right to counsel = higher standard to waive to show it was an informed
waiving
 Lack of maturity, if police violated = auto exclusion
 Created with DP in mind to go from factual to legal guilt with appropriate checks
and balances
 Protections in place (i.e. right to counsel under 10(a)(b)
o 3 steps (informational, holding off, right to counsel)
 S.11 requirement for youth and absolute right to public counsel
 Public hesitation looking at it as a “get out of jail free-card”
 Queen and J(JT)  charged w/ murder and police obtained
incriminating statements which violated requirements under YOA
o Creation of concern of “street smart teens”
o Dube stating that rights and protections under YOA would affect
rights to victim and CC model
o Creation of higher standard = end of interrogation for young
persons
o Public’s perception of this decision = young person laughing in
face of CJS and letting them go on these crime sprees w/o
consequence
o What the media missed = while statements were inadmissible, he
was still conviction of manslaughter, just not for offence of
murder
o Was DP for CC
 Did the YOA promote DP rights and model or did it result in a more CC
approach?
 Some argue that the YOA = classic example that DP resulted in a more
CC oriented system
o Punishing the worst cases
 YOA increased punishments for the worst cases
 Public protection used as instrument to (+) transfer of young persons to the adult
system
 deterrence was used as some of the primary objective of sentencing
 can be used to explain how DP is for CC
 one positive aspect of the Act was that it created alternative, use of sanctions and
alternatives as a glimpse of a DP approach
- Alternatives
84

o Risk and information


 YOA allowing for release of information to identify high risk youths
 Done to prevent criminality down the road
o Crime prevention
 Using non-punitive measures as a types of punishment
 Using treatment as a way to prevent crime from happening
 Crime prevention and using treatment while looking at underlying causes
 Target at risk youths
o Family conferences
 Creation of tools which include family intervention
 i.e. counsellors, parents etc. and is most similar to non-punitive approach
es and similar to sentencing circles
- The YCJA
o The need to new youth justice legislation: the YOA
 System lacked a clear philosophy to why there is a different justice system for
young persons
 Incarceration rate skyrocketed under the YOA, Canada becoming the highest per-
capita incarceration of young persons
 Recognition that the court system was being overused for minor cases that could
have been dealt with via outside of the justice system
 System did not make a clear distinction between serious offences and less serious
offences
 System didn’t give sufficient recognition to concerns of victims
o Preamble and declaration of principles
 S.3 was the primary source of the principles to guide the decision-making process
under the Act
 Served as a guiding mechanism for how the sections were defined
 YCJA contained the principles and objectives of the youth CJS
 YCJA  focuses on the interests of victims, reserve incarceration for the most
serious offences, prevent criminality, aim to rehabilitate, to reintegrate the young
person back into the community, ensure meaningful consequences, reflect the
fact that young persons lack the maturity of adults, measures of accountability
need to be consistent with the levels of maturity for young persons
 Interventions have to encourage repair of harm that has been caused
o Extra-judicial sanctions
 Studies have shown that extra judicial responses are effective
 Allows for the young person to repair what was done to the victim and the
community
 Increasing these responses creates more resources
 The YOA permitted use of alternative measures but provided little guidance to
when those measures were appropriate, the YCJA addresses this
 Extra judicial measures only used when young person admits responsibility to the
offence
o Conferences
 Under s.19 of the YOA
 Any actor within the CJS can convene a conference with a young offender, and
any person connected to the young person can participate in the conference
 Can be a restorative mechanism for the young person
 Ensures that the young person received meaningful consequences for what was
done
o Pre-trial detention
 When YCJA initially came out, could not detain a young person in custody
85

 Presumption was that the young person to be presumed to be granted bail if there
was no likelihood that they’d be found guilty of the offence
 If a judge holds a child in custody while awaiting charges, must consider
if anyone can take the child in custody while awaiting a disposition
o Youth sentences
 YOA = no clear outline for sentencing
 Highest incarceration rate
 Receiving longer sentences than adults for the same offence
 Over 80% of custodial sentences given for non-violent offences (theft,
property damage etc.)
 First time offenders given jail (In ON, 8% of first timer offenders being
given jail)
 YCJA = specific and clear principles for sentencing and meaningful consequences
 Sentence cannot be more serve than what a adult would receive
 Sentence must be similar to what other youths are given in other
jurisdictions
 Had to be proportionate to the severity and seriousness of the offence
 Sentence must promote rehabilitation for the victim and the accused
 Young person could not get jail sentence unless they: committed a
serious and violent offence, failed to comply with other non-custodial
sentences, violent indictable offence and has a history
 Creation of tools that can be given as a sentence to a young person
 A “Stern” lecture or talk from a judge or person of authority
o Probation, intensive supervisor order, deferred custody and
supervision (house arrest), intensive rehabilitate custody and
supervision (for most serous)
 YCJA in 2002 and 2013, most significant changes made to YCJA and
much more punitive in nature and further from the intention and initial
aim of the legislation
- Response from Conservative government  re: changes to the YCJA
o What was changed:
 Introduction of Safe Streets and Community Act
 General principles of Act were amended, protection of the public was
highlighted as a goal for the YCJA, changed the laws as it pertains to the
bail process (focus on young persons charged with serious offences or
pattern of criminality), expanded the definition of what a serious offence
is (public mischief, theft over 5k, dangerous driving etc.)
o Some where max punishment was 5 years or adult disposition
 All done to ensure violent or repeat offenders will receive sentences that
reflect their criminal activity
 Definition of violent offence changed to “activity of danger that could
harm another person by creating harm”
 Requirement for criminal activity now including cases where
extrajudicial measures used
 A young person over age of 14 who has been charged with murder, sex
assault must be considered an “adult” and charged as such
 Publication of young persons identity
o Name shielded via publication ban, old provisions under limited
circumstances where can be lifted
 Change the record keeping requirement for youth
86

o Must keep track of when extra judicial measures are imposed for
youth
o Referrals to be kept track to show pattern of offences being
committed
 These changes were indicative of a more CC response to criminality, much
different from Liberal approach in 2002, displaying the swing of the pendulum
 How we’ve dealt with young persons in system from coercive CC
assembly line under the JDA to development of YOA where D protections
in place, but still resulted in a very CC oriented approach in which
custody was overused and reliance on punitive measures. The YCJA
which was more DP oriented in nature and tried to deal with issues of
over incarceration, and subsequent amendments made to increase use of
incarceration against young persons
o Number of significant changes overtime re: how we deal with
young persons

Lecture 7 Minorities
1. Introduction
- From perspective of accused and victims and potential victims
- Packer’s theories and one of things he theorized = rise of DP would help minority
groups in society
o In 60s = sig time of racial violence and discrimination and believed that DP
would restrain the state (checks and balances on the state); Packer and these
checks and balances helping those in minority
o Focused on acts (activities by officers routinely involved in investigations of
criminal offences from source of civil liberty violations esp. by those in
minority groups)
 Focus = move from how law operates in individual cases to specific
outcomes in criminal processes
o Stats = despite DP protections, still seeing over rep. of minority groups in
prison populations and in custody
 US  over rep of blacks in system
 Canada  over rep of Indigenous people in system
o Critiques of P’s theory = DP model may respond to racial discrimination in
individual cases, but not changing outcomes in other criminal cases in courts
 Run of mill cases = majority of cases within system
 Having rights is great, but sometimes more costly to invoke those
rights than to wait in custody pending their trial
 Many obstacles in course, but these checks and balances not
really being utilized
o Fairness is there, but it really isn’t there
87

o Danger by having DP protections that you are


legitimizing the system and the legitimization of the
criminal sanction
 Minority groups over rep as victims of crime (POC, indigenous,
LGBTQ+)

2. Minorities and Due Process


- Result is symbolic instead of having effect on system, just persists as status quo
- Use of criminal sanction for victim is through this chapter, similar to women and
children
o Criminality against certain groups often used to invoke a CC approach and
reproduces assumption that criminal law can deter crime and offences
o What happens when DP fails, CC fails, punitive
 Victim non-punitive
 Trend towards dealing with them via non-punitive manner and nature
o Canadian courts slow to recognize DP decisions having implications to
minority groups and persons out there to protect
a. Investigative Stops
o S.9 and dev of that law over time, and in ON HTA allowing random stops of
motor vehicles regardless or reasonable suspicion of a legit highway
regulation
 Case law Hufey and Manninen
 Police power could be abused by officers
 Too much discretion (how they look, color of skin)
 Rife for abuse because of powers
 Jurisprudence developed as for how police handle their powers @
stops
o From perspective of person who would receive these stops
 Study
 Erikson study
o Findings: police often engaging in “constant proactive
steps of policing”
 Probing and looking and maybe acting in ways
to further these hunches, beliefs, and suspicions
 These proactive steps often used to remind
marginal ppl dealing with or order of things
 Western Canada and proactive policing often
involving Indigenous persons more than others,
Toronto and overuse of stops and detentions for
POC’s
88

 Commission for systemic racism in ON


o Findings
 Interviewed ppl, asked how many times stopped
by police in 2 years
 45% of black males = yes vs 25% of white
males and 23% of Asian males
 “why do u think police stopped u if it wasn’t for
a criminal activity”
 Black males police associating them
with criminality and trafficking, being
outside of societal order, with individual
not of a POC
 Huffsky decision in SCC and concern of these
investigative stops
 This authorized RIDE for traffic reasons
under HTA
 This type of proactive policing allowed
because this stopping was rationally
connected with increasing safety
 Ladouceur dissent
 Minority being last straw and allowing
police to stop persons at ay time and
subject to significant abuse by police
officers
 SCC if police are caught doing this
via misuse will face consequence of
having that evidence excluded
 Menellithen
 SCC finding = police abusing powers
given to them, according to Huffskey;
nothing to with HTA issue
 Few remedies to those stopped
 Carding and how it relates to police powers under investigative stops
 Carding = practice where police confront individuals and
gather as much info as possible
 Concerns = individuals ascertaining right to walk away without
detention and often times you would see confrontation
o Toronto example:
 If you are a POC, more likely than not to be
stopped than a white male or female
89

 Officers often stopping minority groups at a


higher rate than white individuals
 What was done
o In 2016, ON gov enacted regulation made under the
Police Services Act (sets out rules and resp. of officers),
dealt with collection of info in circumstances in carding
o What amendment attempted to do = control how police
would engage in this collection of info and that it
wasn’t abused and persons subject to carding would
understand their rights when being carded
 Officer must tell individual that they are not
obliged to give any info and officer must take
clear why they are trying to solicit info from
them
 Regulations placed to ensure duties are followed
under this info gathering
o Post enactment
 Ministry of corrections and safety in 2017
appointed an individual to conduct and
independent review and look into carding if
actually worth it (advantages and disadvantages)
 Conclusion = carding doesn’t work, failure and
only generating low quality intelligence
 Leads to greater alienation between
certain communities and officers
 Roll back to carding = lower crime rates
 Police already having sufficient tools to
deal with crime and violence
 Practice which can lead to misuse by
police
 Little to no value as an enforcement
mechanism tool
 Recommendations
 Gov to take harder line on street checks
to tighten def of “identifying info under
specific circumstances”
 Wanted to see creation of education
programs and mandatory destruction
rules in place
 Nothing of significance has been done
90

If evidence is not found and not charged, few routes you can
take if subjected to inappropriate police actions
o One option = to enact a civil suit

b. Complaints against Police


o If evidence is not found and not charged, few routes you can take if subjected
to inappropriate police actions
 One option = to enact a civil suit
o Solutions are minimal because of adversarial nature
 Money and inability of some to use such remedy
o Internal complaints
 Most cases, these allegations are investigated by other officers
 More often than not, they are found to have done nothing wrong
o Complaint mechanisms
 Smith case
 Jamaican woman partially stripped searched by police and no
charges
 Complaint made and investigated (took 8 mo.) and 3 diff
hearings took place and first 2 collapsed because of allegations
of bias by police force
 After 3rd = found allegation could not be proven due to unclear
and unconvincing evidence (need clear and convincing
evidence)
 Drummond case
 TV personality in TO
 Subject to high risk take down by police
 Complaint made
 Police chief recommend review because of clear violation of
powers
 Complaint in front of board was rejected by board even though
they couldn’t locate the civilian who claimed a black man get
into similar car with a gun
 What we see through complaint process, individuals subject to abuse
of police powers, have a greater ability to bring actions to bring police
accountable through the process and trial via a Charter application
 1990s and DP doctrine and shaping what s.9 was all about all
with view to prevent discrimination against minority groups
 Queen and Simpson
o Used HTA to further an ongoing investigation
91

o Appeal court found it was a clear violation of police


power under HTA and boundaries overstepped
o Charter application brought
o Recognition that these stops and detentions and rife for
abuse
o Exclusion of evidence @ 24(2)
c. Bail
o SCC slow to appreciate DP decisions for minority
o Lack of good stats to highlight to problems and how these powers being
misused against these groups
o Ontario commission on systemic rescue commission
 Results= over rep of black males being denied bail esp. charges
involving drugs and drug trafficking
 Finding = persons social economic status resulting in their detention
o Bail system continues to be challenged in pursuant to the charter
 SCC overtime upholding CC bail provisions in Charter challenges
 515(10)(c)
 Tertiary ground upheld on challenge
 R v St-Cloud
o Court took steps to expand the use of tertiary grounds to
detain an individual
o How these grounds before should be used in
exceptional cases (homicide, sex assault etc.), and other
cases saying that they shouldn’t be just the exception as
long as it maintains the confidence of justice
o Decision finding that territory ground to be used in any
case and widened application of ground and can be
used in any case
 SCC decisions supporting how DP = CC
o Allowing police and judges to authorize pre-sentenced
custody while looking fair
o Upheld of (6)(15), reverse onus on you in these sorts of
offences
 Must show courts onus
 Finding that it as constitutional
 Reverse onus provisions challenged = (d) of sub
(6)
 Survived charter scrutiny because maybe
dealing with individuals who are kilo level drug
92

dealers who have means and ability to escape


prosecution, have safe havens and who the
provisions is in place to protect against
o R v Kersten
 Dealt with if this charter is charter compliant
and found it was necessary due to dealers
having flight and activity
 Acknowledged it would affect street dealers, but
predicted that they would be able to satisfy the
onus
 What commission on racism said was this
reverse onus having negative impact on POC
getting bail
 Reverse onus contributing to higher
numbers of minority groups in custody
 PTC
 What used to be where ppl serving for being proven guilty
beyond a reasonable doubt, large percentage now in PTC and
belonging to minority groups and over rep
 Facilitated by how court has dealt with these reverse onus
provisions and presentenced custody in Canada

d. Jury Selection
o Premise = courts reluctant to admit that juries can be selected in a
discriminatory manner or are capable of discrimination
o Commission on systemic racism
 POC under rep in juries across ON
 Findings = jury system in ON results in an under rep of minority
groups in juries esp. in Ottawa and more often than not being from a
more-white area in Ottawa
o Seminal case
 Parks
 Decided that it was essential for trial fairness reasons to allow
accused person to ask perspective jurors if they would be able
to decide fairly and impartially would be decided via race
 S.638.1 (challenge for cause)
o Allows court the ability to grant permission from party
to ask specific questions from jurors
o In Parks case was a black male who was accused or
murdering a white male in a drug deal gone bad, what
93

he wanted to do via lawyers was to ask perspective


jurors, whether the deceased being white would allow
case to proceed without bias
o Appeal found systemic prejudice in system
o Victims rights approach paying attention to subjective
perspectives
 These questions finding to enhance appearance
of fairness and to weed out those who were
racist
o Being able to ask perspective juror if racist would
eliminate racial prejudice from jury
 Aftermath of Parks in ON, some judges viewed systemic
racism to be specifically a TO problem,
o Wilson decision
 Finding it wasn’t just a TO DP protection, but
applicable everywhere in the province
 What Parks accomplished
 Allowed the defense to ask jurors if they were racist, sometime
when perspective juror is sworn in, done so in front of other
perspective jurors
 You are expecting someone to admit they’re racist under oath
in front of people
 How people will not be open to expose their biases
 Done to try to ensure fairness to ask, “are you a racist” and
“no” qualified them to be on the jury
 Other shortcoming of Parks was that it only affected a small
minority of cases in ON and in CA
o Seen as positive decision and everyone thinks that we
have created this DP protection to ensure fairness for
jurors in minority groups and we are eliminating
discrimination within the justice system
 How many ppl will actually admit that they’re
racist
 How many cases will this actually be applicable
to
 Inverted triangle
o Most cases dealt with via plea
etc. and only small percentage
left of cases that actually go to
trial
94

o Only applicable on indictment


and not all that require juries,
don’t all use juries
o Only small portion are going
before a judge and jury
 Out of those trials, Parks
only has application
where the accused is
Black
 Very small portion of
already small
 Application only affecting small
minority of cases
o Had to go to trial, jury, and need
accused being black
o DP victory in exceptional case of
Parkes, but didn’t disrupt the CC
assembly line and didn’t affect
matters dealt with via guilty plea
and didn’t end racial
discrimination in justice system
or with juries
o Small effect of supposed DP
victory in reality

e. Systemic Overrepresentation
- Overtime what we’ve seen in Simpson and Parks, not done much to deal with over
rep. of POC’s in prison
- Black males = 3% in ON vs 15% of prison admissions in ON
o Some will dismiss this as increase criminality
 Need to focus on systemic inequalities which effect minority groups
which leads to this over representation within the justice system
 DP victories over emphasise the contribution of this enforcement
 We’ve just been focused on the machinations of the criminal
 Only looking at this issue through criminal law and practice
 Still under idea of discriminatory enforcement instead of
underlying deeper societal issues

3. Minorities and Crime Control


95

- Sensational cases and CC ways to handle criminality


- Symbolized in new political cases where DP rights of accused were pitted against
equality rights of victims

a. Police Shootings
o Between 1979 and 1995, there were 16 cases where black males shot in
Ontario, in 6 of those cases they were killed
o Policy response:
 Increase use of criminal prosecution against these officers to deter
future violence against black males
 Problem
 Often unsuccessful
o New political case dealing with accused (cop) vs rights of victims and
potential victims
 Police claiming charter protected interests and communities affected
demanding equal applicability of law as victims
 Only 9 of these cases were prosecuted and all 9 resulted in acquittals
 DP nature of investigation and trial procedure
 Normal for cops to invoke right to silence
 Use of methods in jury selection to get a favourable jury for the
cop via peremptory challenges
 Peremptory challenge = that the defense and crown have where they
can challenge empanelment of juror on case (have limits to challenge,
now limited); s.634
 Reasonable doubt and self defense issues
 S.25 and cop claiming self-defence and you’d see unrelenting
attacks on character of the deceased to portray them as the bad
person to protect their life or life of others
 Prior CC = cops could shoot someone fleeing police custody,
this provision was challenged via crown attorney in
prosecution
o Lines decision
 Crown claimed charter protected interest of
society
 This right deprived victims and potential victims
of their s.7 right and couldn’t be justified under
s.1
 Struck as being unconstitutional
 Response:
 Enactment of 25(4) of CC
96

o Arrest must be lawful


o Flight is taken under arrest
o Belief on reasonable grounds to
protect from current or future
fleeing
o Authorization of force because of
present of future risks of harm
 Sig DP protections in
place
 How these DP measures
are used to defense
themselves against CC
charges
 2 cases where cop successfully used prosecution
 Deane
o Indigenous protester shot
o Officer convicted of
manslaughter
o Received conditional discharge
and ended up with no criminal
record
 Sammy Yatim (only successful
prosecution of shooting of an individual
in ON)
o Killed instantly
o After trial, Forcillo was acquitted
of murder but convicted of
attempted murder
o Crown’s theory, even if used
self-defence section and killed
Yatim, wasn’t justified using
force in the second go-found of
force used and no danger at that
point to officers or other
individuals
 Charged with attempt.
Murder for second round
because he died instantly
when he hit the ground
and can’t kill someone
97

again if they’re already


dead

b. Latimer & Disabled Groups


o Shift in Canada from social rights and requirements to legal protections in
1990s’
o Stems from victimization surveys which revealed high rates of victimization
for disabled person in Canada
 High number suffering assault and sex assault
 Emphasizing criminal law as remedy instead of looking at societal or
social factors which could have led to increase risk in victimization
o Latimer
 Daughter in vegetative state and couldn’t speak, see, eat
 Father eventually took her to vehicle and hooked up exhaust pipe to
hose and killed her via Co2 poisoning and charged with 1st degree
murder
 Novel case because it was portrayed as a mercy killing
 Tried to say he was justified in her killing because of her disabled state
 Convicted of second-degree murder, this was appealed
 Special interest groups of disabled persons opposed appeal
 Latimer argued that this type of prosecution against him
violated his charter right because of status of victim and for
what he intended on doing
o Tried to mitigate his sentence because of her disability
o Concern from these intervener groups that the
perception of value of life place upon a person would
be lessened
o Finding that the killing was comparable to a hate-crime
 Life-sentence was to be justified because of
sentence of the victim
 Argued less moral culpability, court of appeal
saying that you murdered someone who was
helpless and how it contributed to sentence
o SCC appeal
 Sent matter back to retrial, it was because how
the police had investigated members of jury and
not providing this info to defense counsel and
sent back to re-trial
 Found guilty of second-degree murder
 Auto- of life imprisonment
98

 Parole ineligibility from 10-25 years,


after jury trial, they polled what it should
be
 Second trial = jury recommended of 1-year
imprisonment
 Judge followed the jury’s
recommendation
 Appealed and special interest groups
appalled
 Demand of CC approach to use the
criminal sanction to be used in the face
of a DP decision finding that the 1-year
imprisonment
o Sentence overturned and parole
eligibility set at 10-years

c. Hate Crimes
o Concern stemming from studies which show individuals being at risk of
aggression or becoming victims because of sexual orientation or gender
o American study used by parliament to amend sentencing provisions in CC
 1 in 5 gay men and 1 in 10 gay women were victims of aggression
 1/3rd of individual received threats of violence because of their sex
orientation
 Media played up these stats and that hate crime on rise, and
something need to be done
 Use of criminal sanctions to deal with societal issue
 One of things done by parliament in response to these
victimization surveys was to amend the criminal code as it
pertains to sentencing
o 718.2(a)(i)
 Aggravating and mitigating factors must be
considered
o Amendments in 1995 expanded to include sexual
orientation as an aggravated factor
 Increased punishment if show crime as motived
because of sexual orientation
 Gov applying equality to gain more punitive and
CC ends
 Didn’t change much in sentencing because the
ON court of appeal
99

 Queen and Ingram


o Already recognized that a
sentence should be more severe
if motivated by bias and
prejudice
o Parliament basically codified an
already common law approach
o Criminalization of politics and
after crime because of sex
orientation
 Gov defended Bill because less expensive to
deal with instead of addressing social and or
economic issues
 “promoting equality” instead of trying to
find root of issue (social or economic)
 “concern for victims” via sentencing
amendments
o Creation of aggravating factors
must still fall in line with
sentencing proportionately (to
offence and offender themselves)

d. War Crimes
o Promised more than it delivered
o Failed to punish any war criminals and without recognizing Canada’s role in
allowing German Nazis’ to emigrate while continually denying Jewish
refugees during and after the war
 Didn’t do much to help these individuals during war
o Amendments
 Required because dealing with retro-active prosecution issues
 Rely on CC and sanction to deal with Nazi war criminals instead of
dealing with them via extradition or immigration process
 Finta case
 Lawyer tried to exclude Jewish person from serving on the jury
 Anti-Semitic stereotypes persisting throughout
 Acquitted and upheld guidelines
 Court said if your going to prosecute a person for a criminal
offence committed, needed a subjective awareness
o Obedience to superior etc.
 Post- Finta, said easier to deal with Nazi’s via immigration
100

o Feds relying upon immigration system to have


individuals extradited from Canada
o Looser rules from criminal and easier to prosecute and
prove (not balanced beyond a reasonable doubt)
o Victimization and victims and crime control

4. Alternatives

a. Crime Prevention
o More expensive as opposed to reactive policies
o Examples
 Being more selective as to who becomes a police officer ensuring they
have proper training
 Encouraging all to reject stereotypes
 Early intervention
 More resources in community can all be seen as a crime prevention
method
 Selected decriminalization (Packer)
 Lower (street) level dealers
o Diversions to deal with first time offenders

b. Restorative Justice puts people on trial


o Via re-integrative shaming
 Community dealing with problem in their own unique way
 Not usually warranted in cases committed against minority groups
because of their minority statuses
 i.e. civil action, but like method of criminal process, is adversarial,
focuses on past an puts people on trial
o Carding inquiry and focus more directly on issue of racism
o Complaints against police (and short comings)
 Still legalistic type hearings and sanctions in place allow cops to deny
responsibility
 Result in officers not wanting to admit guilt or responsibility
 Must be willingness of victim and accused to participate
 Need buy in from all actors to be successful

Week 8  Aboriginals
1. Intro
2. DP and Aboriginal People (as accused)
101

- Bear 2x disadvantage of being sig over rep in imprisoned ppl as victims and potential
victims
- Over rep stressed via examinations and inquiries of CJS and process
o Made real via personal treaties within the system
o No premium to system (efficiently), importance of offender and victim in
these acts and offender and community need to work together to become
whole again
o A solution which can maybe be proactive and addresses abuse via victim/
offender
o These circles offer the most developed and inspiring alternatives to linear
process used to in CC or DP models
o DP and aboriginal persons as accused
 DP can respond to Indigenous cases of abuse and discrimination, not
relevant when accused person ends up pleading guilty
 Last week, Parks decision dealing with attempt by courts to deal with
discrimination in process and use of Parks Question to find ppl who
hold bias against minority on trial
 The decision in essence gives some protection to accused
within jury but sig shortcomings; only applicable to some cases
 Good in those cases available
 As a whole, not applicable before the courts
 Most cases in Canada, ppl are pleading before even getting to
trial
 Rates of our prison pop in Canada; even since Charter, prison
rates have continually increased. Increase in incarceration
while violent crime rate decrease, but continued increase of
Indigenous person over rep in justice system
o Suggesting that DP is not inconsistent with increase in
crime control.
o DP decisions provoking CC responses and can
legitimatize a CC system because belief that the
protections in place will be used by those in CJS, but
not true because of ppl resolving charges via plea of
guilt
 Youth lecture
 Anunga rules, give special DP protections to ppl within CJS,
but we are seeing that despite these protections, these ppl are
being punished more frequently and severely overtime
a. Donald Marshall Jr. (wrongful conviction)
o Concern @ Indigenous people within our justice system
o DP and CC and difference in approach to presumption of innocence
102

DP prides on 10 guilty person go than 1 innocent person convicted;


inverse for CC
 Goal of DP = avoidance of wrongful convictions
 Goal of CC = wrongful convictions are an apparition and doesn’t
happen in a proper functioning legal system
 In Canada, many of these wrongful convictions
 i.e. Milgard, Mirand, and Marshall (ppl who were factually
innocent were charged with crimes they didn’t commit)
o how the criminal process is not a perfect system
o Marshall
 Miscarriage of justice and happened bc of ways society looked upon
Indigenous persons
 Mi’kmaq
 Sentenced to life in prison for murdering his acquittance in 1971
 Spent 11 years in prison before being acquitted in 1983
 When conviction overturned, the presiding judge placed some of the
blame on Marshall saying that he “was the author of his own
misfortune”
 Public inquiry held to look at his wrongful conviction, panel of 3
judges found that Marshall would not have been convicted if actors in
CJS had carried out their duties in professional and competent manner
 Many professionals criticized for not following DP standards
of fairness for the accused
 This happened Pre-Charter
o Pre-Stinchcombe and right to disclosure
 The crown in this decision was criticized withheld prior
inconsistent witness statements
o These were contradictory and coerced statements that
were called at trial
o Imp part of criminal process
o These inconsistencies are the basis for the defense
lawyers and being able to challenge on these
discrepancies (2 pillars to determine what a witness is
good = reliability and credibility)
 Defense criticiser because they didn’t take steps to receive
disclosure
o No attempts to obtain these statements
 Investigation
o Criticized because of improper techniques
o Stinchcombe and s.7 disclosure “the right for Individual
for person accused to have all non-privileged info’s
whether inculpatory or exculpatory)
103

 Marshall referenced in Stinchcombe; lack of


disclosure contributing to this wrongful
conviction

- Lecture 9 Crime Victims


1. Introduction
- Crime victims discussed via diff groups; women as victims and potential victims and
indigenous ppl
- Crime victims shaping law, big impact via way its shaped
- group as victim and victimization surveys role in dev governmental concern for crime
victims
o victims as group made known via criminal law, despite fact they are a less
definable group
o findings often times with these groups = focus primarily on punishment
 increase punitive nature of offences
 how to get more victim participation in proceedings
- creation of special interest and advocacy groups via CC
o victim punitive model = melding of victim’s w/ CC models (w/ some
differences)
- crime victims portrayed as consumers of CJS and of CC services
o focus via governments on wants of crime victims result = more CC oriented
approach for what they perceive for what victims want
o victims as consumers has legitimized a CC approach to dealing with justice
system
o fed gov promoting interests within crime victim
o victim of crime = populous movement since its conception in 90s
o not just fed gov to please victim, but also providences to appease consumers
aka voters
 demands via victim crime groups for constitutional amendments for
rights of victims to have same protections as accused
- result = most victories have to do with punishments and most victories have to do
with changes to punishment

2. Rise of Victims Rights


- Imp of victimization surveys as victims
- Packer and no consideration for victim (and role in CJS)
o Victimization surveys reveal that ppl surprised by how much unreported crime
is committed
o Apx 1 in 4 have some criminal offences taken against them
o
104

a. Victimization Surveys
- Focus on victim on CJS reforms
- High levels of unreported crime = inadequacy of CCC to commit CCC offences
o Parallel of victim’s rights @ US; same in rise of DP of CAN vs US
o Funded via fed gov
 OG intention in Canada was the attempt to ease concern that fed gov
had just abolished cap punishment
 Concerns how society would perceive this @ admin of justice
o Some were apart of a crime prevention study, focus on being proactive instead
of being reactive in nature
 How can we stop it before it can be committed
 Unfortunately, these studies by early 80s focusing exclusively crime
victims and finding out what crime victims really want
 Connection between crime victims and persons who vote and elected
representatives, they began to be treated as consumers
 “the customer is always right”
 In line with gov’s approach with crime victims and victims
 Victims defined as new consumer in CJS
 Personal and emotional feelings of those crime victims became
politically salient
o These anecdotal cases arisen @ sensationalized stories
and backlash = what politicians wanted to take
advantage of when trying to appease these consumers
 Surveys revealing startling figures @ how much crime is out there and
helped back up claim that crime = serious problem in Canada and
crime prevention (despite crime rates decreasing)
 That crime wasn’t a problem or issue/ big deal
 1 in 4 at some point fall victim to a criminal offence
 Significant amount of unreported crime and consistent across
observations across various groups
 A lot of ppl out there who don’t use the CJS
o Some say some victims don’t report crime against them
because they dealt with it in better way than CJS and
better options than what would have been given
 Dealt with criminality themselves
 Once mater involved in CJS, the victim loses
control via how case proceeds because there are
professionals taking over case going what they
believe is best (from micro to macro)
105

o Crime is too minor


o Lack of faith in police, not only investigation, but how
they deal with the victim
 Victims employing own measures outside of
CJS to deal with criminality
 Taken as a sign by some that higher
levels of unreported crime = CJS reform
must occur to bring unreported crime
down to zero
 Politicians construe this to reform CJS to
encourage reporting
o Their view = no crime in society,
and for this = must be full
reporting
o Criminologists = unreported
crime may reflect empathy and
tolerance from society
 Contrast of ideology of
victim wanting eye for
eye
 Surveys used to push CC responses
despite signs that most victims didn’t put
priority on punishment or state
intervention when dealing with criminal
acts committed against them

b. Task Forces on Crime Victims


- Seen in both US and CAN to deal with crime victims
o Canada
 Born out of gov’s interest post cap punishment abolishment and
wanted to know what society thought about the CJS
 Wanted to know if crime getting worse
o US
 Born out of populist movement
 Focus primarily on victim punitive approach, vs in Canada, we started
looking at issue of victimization via non-punitive victims’ rights
approach
 However, as time has gone on, that approach in Canada has
changed, moving to a move victim punitive approach, similar
to how they started in the US
106

Seems although over time, the CJS has become Americanised


via DP revolution after enactment of charter and politics of
victim’s rights in Canada
o Focusing on Canadian task force
 Emphasis on needs of the victims, rather than the rights of the victims
 Initially no opposition to the fact there are special protections
given to accused
 DP integral part of CJS and fundamental principles of CL that
must be protected which must be recognised
 Cannot override rights of accused and respect adversarial role
of state and disproportioned power imbalance and Charter
rights there to ensure protections
 1990’s this approach changed
o Became a zero-sung game between victims and accused
o Task forces recommendations to amend CCC with
heavy emphasis of rights of victim vs what they need
 Recommendations by task forces to amend CCC
 Judges to consider the concept of restitution
o If victim undergone $ loss, judge to consult tools from
CCC, they not only to sentence them, but also order a
restitution order to cover costs from victimization
 Victim impact statements
o Tell judge how crime has affected them
 Publication ban
o And increasing use of these tools so that victims
wouldn’t be identified
 Taxing system
o Where you’d tax the individual of criminal activity
o Taxes on top of fines to accused persons
 Some of the tools championed for victim and for accused
 Restitution
o Not only to benefit victim, but for accused to appreciate
wrong, and promote responsibility and rehabilitation as
$ to give back to victim for criminality to help them
with re-introduction to society
 Sentencing tools from victim to accused born out of these task
forces
 Treatment of respect and supplied with more information than
previously given to them
107

o Previously detatched from themselves and what they


wanted
o Detachment of victim from CJS and process
 Task forces in CA
 Didn’t examine restorative measures or proactive measures
(addressing criminal activity before it was even committed)
 Short comings
o Focus exclusively on what victims wanted

c. Victims Bill of Rights


- Under the umbrella of politicians looking at victims as consumers and people who
vote
- VBR rep battle between victims’ rights and accused rights
- When victims’ rights are pitted against DP rights of the accused, often times, the
victim will lose
o Protections enshrined in Constitution for accused
o Many principles of fundamental justice for accused
o Going to lose most of time because charter = supreme law of land in CA
- Greater amount of state intervention needed; courts hesitant to do so
- Initial VBR = attempt to bring victim closer in CJS
o Want victim to become more informed
- Around time of VBR in Crime and UN declaration demonstrating progressive
potential for victims’ rights
o Contemplated positive state action in order to protect the integrity of the
victim, promote respect for victim and gave wider definition to the concept of
what a victim is
 Same as discussed last week in Aboriginal justice
 Victim not just criminality committed against them, but also
community, family, friends, other individuals
o UN placed this policy of dealing with victims within context of social policy
 Attempt to deal with criminalization of politics and have to look
beyond scope of CCC for how to deal with victims
 Encouraged restorative justice as a means to deal with victims and
victimization in society
 Did not focus on criminal law professionals, rather, said to take a
holistic and multi-disciplinary approach outside of the CJS or the tools
within the CJS
o Canada in 1988 via becoming a signatory, agreed to basic principles of justice
for crime victims
108

 Reality = focus on criminal justice matters of how to deal with


victimization instead of holistic and mufti-tiered approach
o Principles
 Minimization of re-victimization of victims in CJS
 Treat them like consumers and provide them info of progress
and process
 Give them options
 Victims to express their views and convenes of case when
appropriate
 CC approach affirmed via final principle = victims have duty to
cooperate and always report
o The exact opposite of what some believed was a proper
approach and letting victims deal with criminality only
their own unique way and only when they want to
engage the state should the CJS step in
 Example of provinces stepping in on this and being tough on crime
 Opposite approach would be to say we need to ensure robust
system for accused and protections for victims (this won’t win
a vote)
o Many wont take a soft-on-crime approach
o “hug a thug”
o Tough on crime and changes to CJS win votes
 First province to introduce victims bill of rights = Manitoba
o Crime victims defined as being consumers of CJS and
defined them of being victims of the accused’s DP
rights
 The opposite of initial approach in CA
 Saying zero sung game and battle
 CC focus on dealing with victims when they fell
prey to criminality
 Focus on informing and providing info to
victims
o Differences between Manitoba vs Ontario approach
 Manitoba included alternatives to criminal
prosecutions
 Gave victims info from use of possible
restorative methods
 Created victim assistance programs and
committees
 Ontario = CC focused
109

 Clear that victims’ rights and concern of


that = not intended to disrupt the CC
assembly line of criminal justice
o Gov may make these changes, but changes to the actual
assembly line system do not change
 Victims are given rights, as so far as not
interfering with assembly line
 DP and same with victims’ rights
 Limited to amount of resources
o Say that changes ought to be made but no funding
towards these programs
o Before, only option to initiate civil proceedings
- Demonstrate privatization of victim’s services, esp. in ON

d. Canadian Victims Bill of Rights


o Sig changes = fed statute of Canadian Bill of Rights
 Elections: conservative gov ran on a platform of being tough on crime
 At same time of violate crime rates going down
 Conservatives enactment of Bill of rights
 Royal assent in 2015
 Made sig changes to CJS pertaining to dealing with victims
o Canada evidence act amended, previously gave spouses
right to testify against other
o Under new Canada evidence act, spouses can be
compelled to testify against each other, in exception to
the rule
o Bill also changed amount of info a victim could get,
allowed for victims to request and get court documents
 i.e. release conditions for bail, correctional plan
details, work release info
 also, can ask for more recent photo of accused if
available
o expanded use of restitution orders, wasn’t just limited
to cases with actual loss and application of those even
not if tangible in nature
 allow for them to act on these orders as if they
were a judgement
o victim impact statement
 could be given outside of court or behind a
screen
110

o req of court to notify victim of any plea agreement


between crown and accused
 previously only ON and MAN having such
guideline
 ON = any pre-trial plea
 MAN = discussions between crown and
defense @ charge
o Created new rights for victims in the process
 Right to information, protection (safety and
security of victim throughout proceeding), right
to participation (victim impact statement and be
informed and participate in plead bargaining
procedure), right to seek restitution nd
protection against state actors (punitive VR
approach because state actor are held
accountable because if not doing job, then it is
the victim who’s affects)
 Rights must be reasonable
o Cant endanger life or security or
prosecution
 All done in the name of the victim
 One of concerns of affect of admin of justice is
focus on concept of plea bargaining
 Before dealing with Crown
 Now
o More apprehension now as to
what the victim may think @
plea bargaining deal reached
o Concern from Crown perspective
that if victim doesn’t agree or get
them on their side, complainants
may be field against them
o Victims now explicitly or
explicitly causing issues because
may not take certain positions
and potential backlash and job
security of may be perceived in a
certain way
o Crown getting input from victim
111

 What can get in the way


of appropriate resolutions
of trial and beneficial for
victim, not just focusing
on punishment
 Encroaching on plea bargaining, and even CC proponents
concerned about
o Plea and resolution before trial = imp, now these bills
of rights giving victims ability to interfere, or creating
speed bumps
o

3. Victims Rights in the Criminal Trial (all of these are dealing with the punishment of the
offender, that’s the commonality between them; all deal with procedures which happen
after the person has been found guilty)
- Often that victim’s policy on sentencing = punitive

a. Focus on Punishment
- Punitive and contribution to the criminalization of politics in Canada
o Dealing with victimization via CJS, instead of UN multi-disciplinary ways of
dealing with it

b. Victim Impact Statements


- Where victim has opportunity via written or oral submission to provide court with
details of effect of criminal act has had against personally, others, and damage/ harm
caused
- Well known case dealing with this = Antler (1982)
o Victim attempted to make submissions at sentencing heading
o Refused right to do so because finding that victim had no standing within
CCC
o Court expressed concern that the victim impact statement could result in
arbitrary sentencing because popular idea that victims were too emotional and
irrational to deal with the punishment of the offender
- 1988, and sig reforms made to the CCC and victims’ rights package which created the
concept with is the victim impact statement
o Allowed victim to provide statement in writing
o Not allowed opinion on sentencing g
o Judge would read out, judge didn’t need to consider it
o Any issues with facts in statement, the normal protects applied to accused
112

 Crown would have to prove what victim saying beyond a reasonable


doubt
 Subject to potential cross examination
 No opposition in Parliament
 Not initially a major CJS issue in CA
o Stats
 After this section was enacted, few cases has VIS filed
 Low rates of participation because of subject to being cross examined
 Concerns that the courts or actors not taking proper steps to secure
statement from victim
 After 82, allowed to appear at parole hearing
 Happened in same year that fed gov stopped funding victim’s
compensation schemes
o Said one thing but did another
o Took away funding to cover losses
o Over time, the statement has evolved
 Comprehensive events
 Court #7 and impact statements
 Forefront of every sentencing
 Victim impact statement now is a tightly controlled process with
intertwined rights
 Since 88’, amendments made requiring courts with reasonable
time for victim to give their statement
 Power of adjournment ot get these statements
 New form to sue of these statements
 Presentation (photos of themselves before commission of
offence)
 Now court must take into consideration (method of excluding
victim from process)
o S.8 to be read in conjunction with s.1
 Creation of community impact statements 722.2 to be
considered in a sentencing hearing
o i.e. the sentencing of drug dealers in the market from
the businesses and impact held on the community
 some parts may be deemed inadmissible (irrelevant within
statement)
o not to be used to criticiser the accused
o not to argue or re-assert facts
o not there to recommend a sentence
113

o not there to seek personal revenge on an accused


o no diagnosis to be made of accused
o cannot mischaracterize the offence within the statement

c. Faint Hope
- Undergone sig change in CA, conservatives got rid of this
- It was a tool within CCC where individual sentenced to first degree with no chance of
parole after 25 years, allowed accused to apply for chance for parole after only 15
years
o Judge to consider merits of application
o Doesn’t mean person would get parole or chance, they just had the chance to
apply to get a hearing before the parole board
 Seen as something that had adverse reactions
 Court ruled that VIM not admissible in faint hope hearings
 Adverse public reactions
 Amendment for victims to
 Rationale of victim’s rights overtime
o Faint hope becoming more populist
o Changes made to hearing because of the concern
o Gov applying screening test for who could apply
 Victims rights emerging as unforgiving and emotional
 Why to keep it
 Encourage rehab
 Safety of those within correctional system
o If someone can’t apply for 25 years, where is the
rehabilitation and what harm placing guards and other
inmates in
o What stops person from committing such acts within
the prison system
 Like a release valve to handle pressure within correctional
system
o Then what expectation can you have to be better
 Not a lot of faint hop cases in ON
 More attention @ Clifford Olson cases
 Prior to amendments, person would have victims come every
year
 Re-victimization
 Small cross section of all victims in society
 Faint hope as re-victimization exercise
114

 Screening process
 All lost sight to people eligible didn’t apply for one
 Loses sight that anyone in CA released after a faint hearing,
has never killed again
 Repeal of faint hope clause
 Bill c-46, given royal assent repealing faint hope from CC
o Anyone guilty of murder after Dec of 2011, no one
would be able to apply to parole edibility before
o Safety valve = removed
o Parole vehicle to promote restorative means is gone
 Process to get persons who committed murder
prior to 2011, application has changed as well
 Needs substantial likelihood of
considerations, and limit to times a
person could apply
o Form 2 now 5 years
 Major changes to judicial review of faint hope
under 746

d. Victim Fine Surcharge


- Seen via gov as way to promote rehabilitation and resp with offenders
- Tax placed upon individuals guilty of criminal offences
o Example: person committed offence punishable via summary and guilty, also
subject to $50.00 victim fine surcharge
 Indictable = $100
o If given a fine, additional tax on top of sentence
 Judge retain discretion to waive victim change surcharge when unfair
if person didn’t have means to pay for it
 If outstanding charges, may not be able to renew drivers’ licence,
pardon
o No guarantee via gov that the money would actually be spending on victims
or victimization programs
- Conservative gov
o Wholesale changes
 100.00 of summary, 200.00 on indictable
 Person committing various theft etc. where pleading guilty in a sitting
and facing many of those surcharges
 Increase 15% to 30% tax on i.e. under the influence and over
80
o Got rid of judicial discretion to waive victim fine surcharges
115

 Many debates and conflicting decisions


 Debate on whether the punishment was constitutional
 Charter challenges saying it was cruel and unusual
 In Michael Ontario Superior found it unconstitutional, court of appeal
found it to be constitutional
o After lib gov elected number of criminal justice reforms and pragmatic
changes to CJS
 Suggested amendment to CCC allowing judge to waive this
 Went on for 3 years w/o any changes
 SCC in 2018 found victim fine surcharges as unconstitutional
 Struck down entirety of section
 After 2018, no courts had the scope for this
 Liberal attention and enacted new legation to deal with victim
fine surcharges
o S.737, amount of surcharge is same and exception
where court on application can pay no or educed
amount if it would cause undue hardship or [….]
 Defines hardship
 Judge now must give reasons for waiving on
record

e. Restitution
- Seen as sentencing tool available to judges to allow restorative justice
- These restitution orders treating offenders as humans]
- In 88’ amended and under 737 enacted
o Challenged to SCC as unconstitutional Zelensky, court found that the CC
provision survived constitutional challenges and found it to be appropriate
when used with restraint and caution and used in more simpler cases
o Grounds expanded in 1995
 Still had to be dealt with in civil court if someone not paying order, the
criminal courts not forcing someone ot pay order
 Continual changes to legislation which makes it widely used and
implemented
 Before if could not afford, didn’t need to pay
 Damages don’t not need to be $
o S.737.1
 s.738.1, diff forms of restitution they can be
forced to pay
 s.739.1, financial means doesn’t prevent court
from an order
116

4. Alternatives
- To victims
o In ON, you can apply to the criminal injury’s compensation board
 Allows for victims of violent crime to insure $ lost from criminal
offence up to 2500.00
 Allowed to victims of violent crime to get assistance for harm
caused
 CC requirement included within this compensation scheme
from those who didn’t report and denied compensation to hose
deemed no to be deserving
 Still continued to process victims in a bureaucratic fashion
 Studies of victim’s compensation scheme and 8-10% of those
eligible seek compensation, and what we see= most people
make claims through insurance companies than through this
injury board in ON
 Example of how compensation and restitution need to be effectively
merged as tools
- Use of civil litigation
o Increasingly used in 1990s and 2000’s by victims of historical sex assaults
 SCC encouraged this type of litigation to go after individuals for
“aggravated and punitive damages” (damage wards on top of pituitary
loss that speaks to denouncement of activity)
 ON and limitations to litigate (ON and 2 years)
 Limitation periods for those who were minors and no limitation
applicable when a minor; limitation period effectively waived
 Critique = adversarial in nature and egregious at best and costly
 Modest if minor role in crime prevention
 Civil litigation is only as good as how much $ defendant has
(short coming)
- Victim reconciliation program in ON
o Collaborative justice program in Ottawa = example of this
 Where offender meets with victims to make amends for crimes
committed
 Emphasis on agreement of facts and feelings and very informal
process
 Non-punitive model in nature
 Emphasises common community
 Offender recognizes crime committed and harm caused
 Victim has power is satisfied and if they want to continue on
117

 Some victim’s groups didn’t like it because they believed it


cheapened victimization
o Low cost and high satisfaction from victim’s vs
criminal justice process
o Issue = funding
 What we’ve seen time and time again cuts made, now no gov funding
and relying via private funding and volunteers
- Crime prevention
o Individuals encouraged to take steps to protects selves from crime
o Relying on public crime control to deal with victimization
o Community based approaches to deal with crime prevention
 Look at social development for this and looking at it as crime for all
(social, housing, and economic development)
 All help and social development is good because of target of
those at risk
 investing in families and investing in childcare, anti-racist education
ad special measure and prevention when merited all of the aim of
protecting crime before it occurs

 Inquiry criticized his lawyers for not working hard enough to request
disclosure and witnesses
 Rec = quality of rep for indigenous accused to be improved
 Reality = Marshall may have been better off in CC model; in
this system, he would have been able to plead to manslaughter
instead of murder
 The actual person who murdered her was only sentenced for 1-
year vs what Marshall served
 Convection appealed and inquiry faulted the way appeal was
conducted
 Lawyers in appeal argued that manslaughter ought to be left
with jury; factual impossibility when you have someone
convicted and kept pushing for manslaughter despite not
convicting
 Lawyer criticized
 For re-canting witnesses and curtailed it improperly
 DP and accused’s ability to confront and cross examine hostile
witnesses
 Inquiry = if the cross wouldn’t have been improperly
restriction when the conviction wouldn’t have occurred
 Jury
118

 No representation
 Appeal blamed Marshal for testifying and that to wouldn’t
have been withheld
 Inquiry = criticized of appeal and forgot of assumption of
innocence and Marshall explaining why he was innocent
o And interpreter ought to be present
 Tran 1994 and good faith basis
 Escaped criticism from inquiry
o Failure to examine why jury found him guilty, crime to
reveal what occurred during deliberations
 Reason to suspect that they weren’t impartial
o Person using words like “red person” and expecting
more “from a white person” coming from juror
o Good faith basis of if the juror was actually impartial
o Parks type question, might have caught the racist but
failing to understand pre-existing racial stereotypes
o Over rep of persons convicted because of wrongful jury
= not case and most plead guilty because they forgo
this; jury reformation and danger that it would
legitimize the system
 Similar to Parks and creating an illusion of
fairness and no racial bias in justice
b. Jury Selection
o Against law to disclose what occurred during deliberations
o Sequestered and no contact to outside world
o Confidential and can’t be released
 Different than in US
o jury selection in ON
 eligibility = property records and questionnaires sent
 many ppl summoned (i.e. 120 persons)
 given a number and is tossed into barrel
 like lottery system
 first 20 selected and process to determine if they can’t sit on the jury
(i.e. someone who is self employed and financial hardship, can’t read
or understand language etc.)
 dep on case have, may be a challenge for cause (s.638 of CC)
 challenge a juror for cause
 where Parks questions is made available to accused
 persons answer will determine potential bias and ability to try
case w/o bias or prejudice
 Peremptory challenge (s.634)
 Yaccabouci report (2012)
119

 In ON = sig under rep of Indigenous persons eligible for jury


 Issue = not a lot of indigenous persons to serve on jury and
right to be tried in front of your peers
 Look at the recommendations
o Ways to include those persons in jury role outside of
property ownership (i.e. the OHIP data base)
 The questionnaire to be re-worded
 give more realistic period for return of
questionnaire
 call for abolishment of peremptory challenges
o r v. kopenace (2015)
 found accused person not entitled to persons of same race and religion
but to a fair and honest process of jury selection
 question = if gov required to make additional efforts to reach
out to persons on reserves
 that jury will not mend broken relationship between society
grounds and justice system
 repetitiveness not about targeting those not on jury role, about
addressing systemic and historic wrongs
o by doing this may result in problems if perfect
reorientation of all races and religions (majority in this
decision)
 saying you’re not entitled to having a certain
proportion of rep of jury
o Queen and Stanley (2017)
 Jury trial in Sask
 Racial divide in society
 Issue = use of peremptory challenges and these were used to
excuse Indigenous persons from serving on the jury
 Result = 5 men and 7 women who were all white and Stanley
being white
 Acquitted for murder
 Flashpoint in Canadian society
 Decision = liberal response via Bill C-75 changed jury
selection process in 4 ways
o Abolished peremptory challenged (634 is no longer
available)
o Altered challenge for cause process
o Allowed judges to stand aside potential jurors for public
confidence in administration of justice
o Allowed trial to go via judge alone when jurors are
below 10 of them
120

c. Investigative Detention and Over-policing


o Last week discussed carding and similar to this
o Case study = JJ Harper case
 Indigenous person killed in investigative stop via Winnipeg police
 ID was asked and attempted to leave
 Const. grabbed him then he was fatally shot
 Last week and dangers to carding and relying on their rights
not to subject to these questions
 Case study for what can go wrong in these interactions
 Inquiry conducted that found the officer shot him got up in excitement
of chase and had no reasonable or probable grounds
 Detention was inappropriate and exceeded arrest powers and
violation of s.9 stop someone
 Const. was trained but didn’t follow basic legal standards
 Simpson and reasonable suspicion to
 S.9 creating police power short of arrest (created common law
grounds of reasonable suspicion)
 DP standards only apply when violate and proceedings are initiated
and don’t hear about it until charges filed
 Harper apart of larger over rep of enforcement action and subject often
to improper stereotypes
 Winnipeg = 3% of pop and 27% of male and 70% of female
(Report inquiry)
o Endorse of community policing to adopt to Aboriginal
culture and standards
o Problem = community policing still having over rep
and police in direct contact to these persons
d. Strengthened DP: Anunga Rules
o Similar to YOA and encompassing more DP protections for young persons
o Recommend that Canada to look to AUS and Anunga rules and pertaining to
interrogation tactics of interrogation suspects (similar what’s in YOA, higher
waiver standards etc.) where state has detained and to interrogate Indigenous
person
 Rules = to call friends or counsel and they can be present
 Clear cautions
 Use of interpreters when appropriate
 Holding off if person was ill or intoxicated
 After enactment, little improvement @ overrepresented
o Despite these measures, no decline in Indigenous
persons in CJS or incarcerated
e. Individual Abuse to Over-representation
121

o Increased DP may protect some abuse from state abuse and actors but will n0t
rectify over rep in prison populations
 Winnipeg example
o Explanations
 Youthfulness of communities, crimes in communities, residential
school legacy and social and economic discrimination
 To public, this over rep was attributed to discrimination in CJS
 Call to reform CJS to stop the discrimination against these
people
 Indigenous Justice offered promising hope
o Criminalizing of politics with women, youth and
minority groups; same thing here
 Changes made are made just to tinker with CJS
and issues within the system are not the be-end-
all
 Public inquires focusing on CJS
recommendations increased of alternative
recommendations
 Over representation being dealt with via CJS
 Cheaper to focus on CJS reforms increased of
more expansive reforms of justice
f. Gladue and Ipolee
o Enactment of sentencing principles (s.718.e)
 Way gov tried to respond to over rep of indigenous people
incarcerated
 Encouraged judges to apply restorative principles for what a fit
sentence was for that individual
o Gladue
 Dealt with what 718 was about
 Something remedial in nature and not a reaffirmation of
sentencing principles
 Found that to reflect special circumstances of the individual
 Judge must consider 2 things when dealing with Indigenous
offenders
o What are the unique or systemic background factors
which may have contributed to their being in court
 Systemic issues and inquires the court looks at
 Poverty
 Overt racism
 Family or community breakdown
 Low income, unemployment
 Alcohol abuse
122

 Residential school attendance


o Judge must look at appropriate sentencing procedures
and sanctions being appropriate
 alternatives
 Probation with certain terms and
conditions, sentencing circles etc.
 718.e to everyone who identifies as Indigenous
 Post – Gladue
 SCC and courts not doing actual Gladue analysis and not being
looked at properly
 Revisited in Ipolee decision
o 718 and what the intention was of government
o Remedial provision and there to reduce
overrepresentation
o Parliament directing judges to specific circumstances
being different than non-Indigenous backgrounds
o Court noted taking “judicial notice” of a proven fact
 History of colonialism, residential schools etc.
(systemic factors)
 These are not CJS factors or matters, this is a
societal issue dealing with the legacy
o Court highlighted issues such as criminalization of
politics and disparities which can lead to criminality
 Not though 718.e to deal with over
representation which are continually ignored by
the government (substance abuse, lower income
etc. and are not properly addressed)
STOPPED AT 1:30:37
3. CC and Aboriginal People (as victims/ potential victims)
a. Helen Betty Osborne (how DP reforms = not enough to protect Indigenous
peoples from being subjected to high rates of being victims or potential victims)
 CJS not dealing with systemic issues and danger in offering CC as a
solution and relying on it = over rep of Indigenous ppl in prison
populations
 Packer’s DP and CC models not offering hope to deal with issue of
over rep
 Alternatives to address over rep in society and potential victims
and victimisation
 One factor in increase in discussion of Indigenous crime is growing
rate od victimization of Indigenous women and children (80% of them
experiencing violence in life, 87% injured and over 50% sexual)
123

 Sexual violence = greatest threat to indigenous persons in


society
 Feminist perspective via violence against women and children
 Aboriginal crime used to express hostility towards Indigenous
people and doubts of self-governance
 Victimization in view of public health
 Suffer disproportionately of suicide, infant mortality etc.,
 Holistic approaches place less emphasis on the sanction
 Some view the disparities in life as victimizations
 Example of Helen Betty Osborne
o She was abducted, sex abused and then murdered
o Charges not laid until 1986 when 4 white males were
accused of murder; one 1 of 4 convicted
o This case along with Marshall led to Manitoba
Indigenous justice inquiry
 Harper = lack of DP protections against
Indigenous persons
 Osborn= failure of CC to deal with criminal
offences against indigenous persons
o After these men charged retained lawyers and refused
to speak, time period when police not bound by the
charter
 Didn’t detain or question suspects
 Some say that this was a double standard
 Manitoba inquiry
 Recommendations from victim’s
perspective
 Osborn case was inappropriate treatment
to friends, family and no support
provided
 Found silence of community related to
general social indifference
 Individual convicted of murder was
sentenced to life after 10 years
o Granted day parole, then revoked
because of violation
 Osborne family stated that they were
happy with his parole despite previous
position. This was changed because her
murder apologized to community and
family to harm caused
124

o Broader social injustices because


Osborne placed in place where
she was because of government
policy requiring her to move to
attend school outside of
community and finding of lack of
social and economic
opportunities which were
contusive to alcoholism, suicide,
and her case, crime
o Led to question if under policing
or under serving of Indigenous
community
b. Under-policing or under-serving
o Manitoba inquiry found under policing of northern areas would not be
tolerated in non-Aboriginal communities
o Situation where expecting indigenous person to use-nonexistent phone calls to
policing which would not be imminent
 Telephones not common place on reserves
 Belief that non-Aboriginal police didn’t care about crime in these
communities
 Belief that may be some value in CC measures in short to deal with
criminality and victimization on reserved
 i.e. overnight detention to deal with intoxication to ensure they
wouldn’t commit criminal offences against others
 justice spending on proactive and not reactive solutions
 worry was adding police would lead to more over
representation
 leads to belief that Aboriginal justice requires more than just
judges and lawyers, requires more healers, counsellors than just
crime control assembly line professionals
o one example of victim non-punitive alternatives are the
use of sentencing circles
4. Sentencing circles (non-punitive methods; because it integrates offending behaviors into
larger cycles in life and attempting to make ppl wh0ole again outside of punitive
sanctions)
o Provide hybrid intervention which promise that discretion within existing
system to be influenced by Aboriginal persons
o Allow offenders, victims, families, elders, and police, lawyers, and social
workers to make recommendations relating to sentencing of aboriginal
offenders
125

 Founded on the premise that accused facing guilt and punishment, the
judge will not make alternative disposition
 These recommendations in sentencing circles often implemented via
sentencing orders such as probation
 If held within CJS, the trial judge can accept recommendations made
from sentencing circle but can also involve some sort of punishment
which is more harsh
 More flexible from the criminal trial because of options
available can be influenced via models of alternative justice
without threatening domination of DP and CC professionals
 Circle breaks down traditional set up of court room and creates
opportunity for increased involvement and responsibility
 Relates to 718.e, other options than the criminal sanction and
prison
 Limits to its utility to decreasing representation of Indigenous persons
within system
 Lack of resources = biggest factor
 In court, not often seeing it used (circles)
 The circles only conducted when resources allow for it, and
even then, sometimes its not available because of time needed
to be involved
 Alternative forms of punishment not pressing premium of
doing git within amount of time and not available in more
serious cases
o Similar to week of women and sentencing of violent
serious sex offences and alternatives not available in
serious cases
o Vulnerable to CC attacks because concern of seeing it
as a soft form of punishment and do not conform to 718
factors such as denunciation and deterrence
o DP concerns of disparity and proportional punishment
 Why should someone in one jurisdiction who
has resources have it over other under funded
jurisdictions
o Unlikely to replace use of the quick guilty plea in
sentencing hearings used to process most aboriginal
offenders through assembly line and utility is minimal
because majority of people plead guilty before moving
through assembly line

5. Aboriginal justice
126

o Aboriginal communities define what they need in justice system and


government requests to deal with community-based crime prevention and
reduction initiatives
o Related to failure of DP and CC to reduce over representation of Indigenous
ppl within the system
o Justice Marie-St. Claire talked about primary meaning of justice in aboriginal
society is that of resorting peace via reconciliation with person
 Indigenous justice incorporates both justice and healing and not
dominated by professionals
 Elders whoa re seen with most wisdom play imp role in Indigenous
justice system, but not there to judge individual and see person as
whole
 Understand their shortcomings and reasons behind criminality
 Focus on harmony, reintegration and rehabilitation of offender
back into community rather just focussing on their guilt
 Sometimes perceived as soft, its clear that appearing before
your elder is a daunting task because they know you, know
family and aware of community you grow up in and see faults
and can call you out on them
o Emphasis of acceptance of own lack of wisdom and to
accept it from your elder to heal
o Also contemplates role of victim as they are brushed aside in CJS for
punishment
 Based on what we think victims want handled via state
 We know that not all victims want what the state wants
 Only when considering victim, offender and community are the
needs served
 Victim defined broadly in justice, those connected in community at
large are also included
 Offender playing larger role in this model than in CC or DP,
take responsibility, accept faults and acceptance is different
than in a court room where they merely accept the facts
 Concern with healing makes offenders past victimization but
not a crutch and the forces for which the offender has given
into
 Aboriginal justice integrated into our own CJS have been
challenged from both the CC and DP
6. Challenges from DP and CC
o DP
 Criticized because of judicial impartiality from either crown or
defense, conflicts with involvement of elder and family and friends
 Concern of self-incrimination and needs to accept their responsibility
127

 Legal representation conflicts with concept of personal


acknowledgement of responsibility and that this takes time and DP
concerned with brining offender to reasonable amount of time and
conflicts with consensus timeline within restorative model
 Principle of proportional punishment concern because different
communities may have different ways dealing with criminality which
can result in disproportionately in how offender is dealt with
o CC
Critiques aboriginal model because seen as being lenient when dealing
with offender
 That the way offender is dealt with is not as punitive as one thinks it
should be
 Aboriginal model is subject to public criticism as not taking crime
seriously
 Ideology of harsh punishment vs aboriginal model
o Not about finding harshest punishment, and can still
take their criminality seriously
o Ottawa, increase in recognition of Aboriginal justice
 Indigenous persons court focused on issues of bail and sentencing
issues for Indigenous persons
 Alternatives but resources just not being there
 Practicality perspective, despite this options available, they just aren’t
used and continue through assembly line dispute others being out there

- Last lecture/ exam prepping


- Conclusionary remarks:
- Moving in Canadian CJS from state vs accused’s rights
 Their rights via DP
 Cases and concepts where power of state was up against the DP rights and
protections of the individual
 From legal rights enactment in charter, it was the case
 At SCC and other appellant levels, were pitting DP rights against
that of the state
 Beginning of lecture of significant imbalance via investigating and
prosecuting individual’s and the rights afforded protections to
accused persons when facing significant state power
128

 “limitless” amount to prosecute individuals and decisions by SCC


to take what was in the charter (legal) and define and create
parameters as to what they mean
o Search and seizure
o Detentions
o Rights to counsel
 From development to enactment in out constitution
and where majority of changes in process and rights
which was in the state and vs accused
o Victims and considerations messing this up
 Political cases, not just battle between state and accused, but interest and
consideration of accused entering into the battle
 Battle between DP rights of the accused and rights of victims
within the CJS process
 Discourse of victim’s rights shaping laws, procedure, evidentiary
rules
 Creeping into the classic criminal case battle of victimless crimes,
women (changes in laws dealing with sex assault, evidentiary rules
with prosecuting and defending these cases, changes via
protections being afforded to victims and not just via cases, but
also parliaments consideration to victims
 Crime victims as whole, the victims and complainants as
consumers in CJS, majority of changes in the CJS whether laws,
procedure or evidentiary rule done in name of victim because of
what is thought what they believe need to be in place for victims
who are now playing large and significant rile in CJS and how
laws deal with
 Amendments that liberals made and future of CJS will depend on
victim’s rights movement
o Battle not just between rights of victims and DP rights of
accused, but how are we going to look at issue of victim’s
rights in Canada?
 Strict victim punitive approach and punitive laws,
mandatory mins, prosecutions and reactive in nature
or looking at it via non-punitive approach and not
just in punishment lenses, but what changes can be
made to how we look at victims
 Victim non-punitive approach apparent in evolution
in YCJA enactment
 Attempts to try and over time take ppl out of
the CJS if possible, to deal with
129

victimization outside of system and


implementation of other holistic means
 Alternatives via aboriginal justice, not
focusing just putting people in CJS and
seeing them go from factual to legal guilt
 Sensational cases like Ghomeshi having impact on prosecution and
dealing with certain criminal offences
o Direct result = changes to sex assault
o Stanley case = changes to jury selection
 Way we look at victimization deals with how
criminality is dealt with
- Themes (relating to essay question for final exam; 5 of them), argument for or against
and some directly relating to some of Packer’s theory
o Rapid rise of DP rights and rapid rise of victim’s rights in Canada
 Seen significant changes to criminal law procedure bc of enactment of
charter
 From purely linear system of trial determining guilt beyond a reasonable
doubt and not dealing with state misconduct within trial, and flipping it on
head and dealing with it directly within
 Charter applications within pre-trial motions and not there pretrial
 Post charter = criminal trial changes and now dealing with issues
of state misconduct within the trial either because of evidence or
procedural protections
o I.e. Disclosure and free from unreasonable search and
seizure
o DP revolution when looking at police powers
 Police investigatory tools being found by courts to
be unconstitutional
 Writs of arrest and no application, now no
respect to charter interests, the evidence
would be useless
 Seeing different DP decisions as defining
what charter actually meant
o i.e. Therens (defining what right to
counsel means and what it means via
detention)
 cop didn’t give right to
counsel and court threw out
breathalyser
 where seeing DP revolution
happening
 flip side post DP decisions is one of 2 thins
130

o either parliament directly responding


via laws that are constitutionally safe
 i.e. search warrants and
seizing blood samples; SCC
sticking samples collected
w/o authorization and now
section authorising tele
warrants, blood seizures etc.
as so long as having grounds
met i.e. reasonable and
probable grounds
surrounding a response
 responses to combat DP
revolution and “police
loosing ability to combat
criminal activity
investigatory methods”
 making laws making it
constitutional as so long as
judicial authorization with
exempts such as if evidence
to be destroyed
 parliament giving tools
within respecting of charter
interests
 seeing SCC stepping back from DP decisions
 Askov and 11(b) and seminal case delaying
with trial within a reasonable amount of
time
 Aftermath = shook confidence of
administration of justice because of how
11(b) being seen as a technicality and
amnesty for criminals
 And how Askov read literally instead of
purposely
 Morin retreat from their own DP decision
and making it harder to order stay of
proceedings, expand time period, making it
a more flex analysis and more onus on
applicant to show some form of prejudice
caused because of violation of right
 Now: Jordan and how it has flipped the
procedure on its head and hard caps on time
131

to go to trial and what was thought to be a


ore straight forward analysis to take place
o Victimization surveys showing criminality previously not
reported
 Triangle of how cases are funneled through the CJS
and only representing reported crime
 Iceberg analogy and significant amount of crime in
Canada
 Doesn’t mean we’re in crisis of CJS failing,
but the unreported crime and focus on
o i.e. domestic violence and sexual
violence then what we thought of,
certain criminality against certain
groups and representation as victims
as a result of these surveys
o provoking victim punitive approach
turning to more CC oriented
response for many reasons
 (1) level of unreported crime
being construed as
inadequacy of CJS, belief
that all crime should eb
reported and that can we
control it

o Rise of the new political case which pitted the rights of the accused vs victims
 Move of state v accused to system where we look at the rights of accused
in opposition to victims or potential victims
 Rise in the new political cases challenging the CJS being a bipolar state
between state and accused
 Many cases of victim’s rights entering into the equation and affecting cjs
 Seen in treatment of “victimless” crimes
 How new political case giving rise to political sanction and based
on CC assumption that the criminal law can actually control crime
o Must believe in deterrence for this to work
o Mandatory minimums and making it easier to facilitate
criminal prosecutions
 i.e. sex offence from rape to sex assault and making
it a hybrid offences which opened up lower forms
of sentencing for individuals and belief in this is
lowering go victimization which hasn’t happened
132

o the criminalization of politics


 seen in many different ways
 where there is a belief to deal with political, social, economic and
cultural problems via CJS system reforms
 rather than looking at the underlying issues which could be causing
such criminality
 present where having parliament responding directly to SCC
decisions
o i. section on indigenous persons and high rates of
victimization and how do we stop that and high rates of
over incarceration for indigenous persons
 Gladue and ipeelee opens eyes to actual issues
causing over rep in prison pop and over rep as
victims
 Not because of how criminal laws formed
 Enactment of 718.2(e) for sentencing of
Indigenous offenders and how to do
 Supposed to deal with over reliance of
prison for Indigenous offenders
 But reading ipeelee, over rep of Indigenous
person is not because of set of cjs, but
plethora or societal and economic factors
leading to such over representation
o Legacy of residential schools, 60s
scoop, remoteness of the
communities to mainstream society,
issues with drugs, prevalence of
drugs and suicide and poverty within
these population; these are
contributing causes to the over rep
within the system and within the
carinal law itself
o Enactment of aggravating factors in principles of
sentencing and crime committed against persons because of
sex orientation
 How gov saw fit what was a societal problem where
person mistreated because of their sexual
orientation and criminal acts against them because
of this and looks at this issue and says we are going
to make it a cc offense but doesn’t change much
because:
 Already a common law aggravating factor
133

 It does nothing; not reason why ppl commit


criminal offences, not because no laws to
aggravate it but underlying societal issues
which plays such a role in bias
o Post charter enactment and DP decisions that followed
produced both criminalised politics in the legislature,
parliament and in the courts system
o DP was actually for CC (packer’s critiques)
 How rise of DP in Canada threatened a CC approach and threatened
punitive factors of victim’s rights in these new political cases, like those in
SCC, but not those in average courts and minimal affect as how how cjs
actually operates
 Enactment of carter and acknowledgment of these protections did not
empty out prisons
 Did not diminish, and in fact opposite is true
 After recognizing specific rights for individual against state activity, our
prison pop has been increasing, this is in the face of decreasing rate of
crime
 i.e. post enactment of YOA, from the State acting in the interest of
the young person and not focusing on the criminal centric model to
deal with criminality and changing it to recognize that young
persons within the YAO have specific rights and protections
against state activity and state interaction
o 180 CHANGE of procedural change and rights which are
more significant to that adults are given
o DP approach and result = highest per capita jailer in world
and was more than US
 i.e. the parks decision
o where a DP decision was made via Ontario court of appeal
to root out racism, bias, and stereotype within jury system
and hailed as significant DP victory to root out bias and
prejudice in the decision-making process
o actual effect = minimal at best because effect only touched
upon small number
o funnel and plea bargaining
o where DP threatening victims and CC at macro, actual
effect is minima;
 DP victories invoked CC responses
 2nd lecture and police and DP victories which struck down their
investigatory techniques and parliament’s response making them
constitutionally correct
 SCC decisions involving CC indirectly
134

o i.e. Stinchcombe in Askov crises


 Stinchcombe = DP victory to disclosure ahead of
time
 result = plea bargaining which is a staple of
CC theory from factual or legal guilt and
becoming an accepted reality and no longer
seen as bad, but as necessary for system to
operate smoothly to take away from finite
resources
 becomes accepted and regulated where it is
facilitated
 Askov did the same thing
 Both dealing with trail within reasonable time and
disclosure facilitated a CC response
 What these victories did provoked a more efficient and legitimate
CC system
o Legitimate = looking at the major cases defining DO
protections to us, looking at it from perspective of CJS and
checks and balances and obstacle approach
o To go from factual to legal guilty = check points in place
via i.e. Stinchcombe and Askov and “perfect procedural
protection”
o legitimizes the fact that at the end of the day, we are more
comfortable to being more punitive to other individuals
o Crime rates decreasing, but imprisonment rates increased
 Reason = belief that system is great with proper
checks and balances, and even if go through system
and is found properly guilty = no more
considerations for wrongful guilt and now you
really deserve punishment and easier to punish
persons if checks and balances are in place
 Same argument can be made for YOA and notion of
fascination with punitive approach
 Often legitimized actual CC system in place
 Can argue that in Canada more CC oriented than DP and reliance
on plea of guilt to make system work to ensure stress on resources
not reaching boiling point to prosecute individuals
o Belief that victims’ rights often = crime control response
 That rise and increase in victims’ rights provoking more CC response (it
is, and it isn’t)
135

 Participation of crime victims and potential victims in justice system were


frequently on side of crown/ state and in favor of increased levels of
punishment
 How many diff victims’ groups looked at called for more harsh
punishments and curtailing rights of the accused person
 Victimization surveys revealing high levels of unreported crime
and construed as means to reform CJS all done w/o considering
how irrelevant the system may be for some victims
 Argument that the CJS may be irrelevant for most victims in
society because of high levels of unreported crime and via making
amends in diff ways outside of going through the CJS
 Back to consideration of risk and harm of revictimization
o Risk is easier to calculate than control esp. through use of
CJS
 Rise in support for non-punitive forms of victim’s rights over time with
less emphasis placed on the sanction while looking at alternatives
 Non-punitive forms of victim’s rights attempting throwing cjs reforms at
complex social, political, cultural, or economic issues or problem; exact
opposite of criminalization of politics
 Attempts to integrate non-judicial actors outside of the classic CJS
and how a victim wants to deal with their victimization
 True
 Parliament and creation of laws and need to be more CC in
response
 i.e. conservatives tough on crime legacy and concern of unreported
crime and victimization and enactment of more mandatory
minimums and creation of more offences when maybe its not what
victims want
 false
 task forces looking at victimization and evolution and what they
need vs what we think they want
o we have a classic view of what they want, and they want
punishment and retribution and want accused persons to be
treated harshly and operate in that way, but evolved
overtime to deal with victimization and other non-punitive
forms to deal with matters outside of CJS; move from what
victims want from macro to what does this specific victim
actually want to go into he cjs, or do they want to deal with
the criminality in some other way
- Prevailing themes
- Victims bill of rights
136

o Canadian victims bill of rights enacted via harpers conservatives creating number
of procedural protections to deal with victims within CJS, other victims bills of
rights enacted at a provincial level i.e. Ontario vs Manitoba approach
 More comprehensive was recently enacted
- 278 and O’Connor (3rd party records application) and reasonable expectation of privacy
o 278.92(1) and Ghomeshi dealing with records in possession of the accused person
o What’s a records? Under 278.1and reasonable expectation of privacy and not a lot
of consensus via what is reasonable expectation of privacy in text messages
- Jordan meant to be clear with ceiling and framework and in 2 easy steps
o SCC didn’t acknowledge delay because of judges, do young persons need a lower
ceiling; things happening all the time
- Final exam
o Worth 50%, 3 hrs
o Similar set up to midterm
o 2 sections
 Essay question (worth 25 marks)
 Going to be similar format to midterm talk about packets theory
and critiques and domination before or after models and talk about
DP retreat ad advance
 Will ask if agree with proposition or theme and ask for things to
support response
 Going to look at essay question and example from police chapter,
one from chapter on women and minorities and ability to draw
upon all materials
 2 essay questions, only need to do 1
 Use as many examples as possible, not just based on lecture, but
also on readings, and materials on CU
 Short answer (25 marks)
 5 marks/ question
 5 of them and choice of 12 short answer (cumulative)
 Questions on major cases, 1 or 2 on major pieces on legislation
(section on the criminal code or piece of legislation like the
Canada evidence act (not), but think about other such as young
persons legislation
o Need to know what section is about, but broader

o

You might also like